N4 Family Midterm Prep U

¡Supera tus tareas y exámenes ahora con Quizwiz!

A G2P1 woman with type 1 diabetes is determined to be at 8 weeks' gestation by her health care provider. The nurse should point out which factor will help the client maintain glycemic control? Vitamin supplements Oral hypoglycemic agents Exercise Plenty of rest

Exercise Explanation: The three main facets to glycemic control for the woman with pregestational diabetes are diet, exercise, and insulin. An individual with type 1 diabetes uses insulin and not oral hypoglycemic agents. Vitamin supplements may assist with helping to keep the woman healthy but not necessarily through glycemic control. It will be important for the woman to get enough rest throughout the pregnancy but this will not assist with glycemic control.

If a delivering mother weighed 140 lb (63.5 kg) at the time of delivery, how much weight should she have lost when she goes home 2 days later, based upon the average pattern? 5 to 9 lb (2 to 4 kg) 30 to 42 lb (14 to 19 kg) 10 to 16 lb (4.5 to 7 kg) 17 to 29 lb (8 to 13 kg)

17 to 29 lb (8 to 13 kg) Explanation: Normal expected weight loss is approximately 12 to 14 lb (5.5 to 6.4 kg) with the delivery of the fetus, placenta and amniotic fluid then an additional 5 to as high as 15 lb (2 to 7 kg) in the early postpartum period from fluid loss.

The perinatal educator is instructing on various emotions commonly experienced during labor. Which complication of anxiety is most important to stress? Shortness of breath Dystocia Gestational hypertension Fetal tachycardia

Dystocia Explanation: Many women experience an array of emotions during labor, which may include fear, anxiety, helplessness, desire to be alone, and weariness. These emotions can lead to psychological stress, which indirectly can cause dystocia. Dystocia is a prolonged labor as the tense woman is fighting against the labor process. Shortness of breath may occur with a panic attack. Gestational hypertension occurs during pregnancy. Fetal tachycardia is not commonly associated with maternal anxiety.

A woman is 20 weeks pregnant. The nurse would expect to palpate the fundus at which location? symphysis pubis between the symphysis and umbilicus at the umbilicus just below the ensiform cartilage

at the umbilicus Explanation: At 20 weeks, the fundus can be palpated at the umbilicus. A fundus of 12 weeks' gestation is palpated at the symphysis pubis. At 16 weeks' gestation, the fundus is midway between the symphysis pubis and umbilicus. At 36 weeks' gestation, the fundus can be palpated just below the ensiform cartilage.

A nurse suspects that a client is developing HELLP syndrome. The nurse notifies the health care provider based on which finding? hyperglycemia elevated platelet count disseminated intravascular coagulation (DIC) elevated liver enzymes

elevated liver enzymes Explanation: HELLP is an acronym for hemolysis, elevated liver enzymes, and low platelets. Hyperglycemia is not a part of this syndrome. HELLP may increase the woman's risk for DIC but it is not an assessment finding.

After spontaneous rupture of membranes, the nurse notices a prolapsed cord. The nurse immediately places the woman in which position? supine side-lying sitting knee-chest

knee-chest Explanation: Pressure on the cord needs to be relieved. Therefore, the nurse would position the woman in a modified Sims, Trendelenburg, or knee-chest position. Supine, side-lying, or sitting would not provide relief of cord compression.

What is Betamethasone used for

Betamethasone is a steroid given to clients to hasten preterm fetal lung development.

A community health nurse is leading a discussion at a health fair for college students on the topic of the signs of pregnancy. The nurse determines more teaching is necessary when the students choose which sign as a probable sign of pregnancy? Positive pregnancy test Fetal movement felt by examiner Hegar sign Uterine contractions

Fetal movement felt by examiner Explanation: Probable signs of pregnancy include a positive pregnancy test, Hegar sign, and uterine contractions. Fetal movement felt by an experienced examiner is considered a positive sign of pregnancy.

A woman comes in for her annual gynecological examination and informs the nurse that she is going to try and become pregnant. She asks the nurse when the best time in the month is to become pregnant. What is the best response by the nurse? "The day after your menstrual cycle ends." "5 days after your menstrual cycle is to begin." "3 days before until 2 days after ovulation." "2 weeks before and 2 weeks after ovulation."

"3 days before until 2 days after ovulation." Explanation: Sperm are able to fertilize the ovum for up to 72 hours after ejaculation, and the ovum remains fertile for a maximum of 48 hours after ovulation. Thus, the window of opportunity for conception is 3 days before until 2 days after ovulation.

A newborn at 1 minute of life is acrocyanotic, HR is 110, is floppy with some flexion, has a weak cry, and grimaces. What Apgar score would the nurse assign this infant? 6 7 8 9

6 Explanation: According to the Apgar criteria, acrocyanosis is scored as 1, HR over 100 is scored as 2, grimace is scored as 1, some flexion is scored as 1, and a weak cry is scored as 1. This totals 6 for the 1-minute Apgar score.

A primigravida 28-year-old client is noted to have Rh negative blood and her husband is noted to be Rh positive. The nurse should prepare to administer RhoGAM after which diagnostic procedure? Contraction test Nonstress test Biophysical profile Amniocentesis

Amniocentesis Explanation: Amniocentesis is a procedure requiring a needle to enter into the amniotic sac. There is a risk of mixing of the fetal and maternal blood which could result in blood incompatibility. A contraction test, a nonstress test, and biophysical profile are not invasive, so there would be no indication for Rho(D) immune globulin to be administered.

Which protective equipment is most appropriate when assisting the health care provider in the delivery of the fetus? Select all that apply. Hair net Goggles Gloves Gown Face mask

Goggles Gloves Gown Explanation: The risk for splashing of body fluids is high during the delivery. Goggles or eye shields, a gown and gloves are necessary for protection. The hair net and face mask are seen in surgical areas.

During the newborn assessment, which finding alerts the nurse to obtain a glucose level? hypotonia temperature of 98.1°F (36.7°C) vigorous crying respiratory rate of 48 breaths/min

hypotonia Explanation: Hypotonia is an early sign of hypoglycemia in the newborn. Other signs include a low temperature, weak cry, and apnea. This newborn's temperature and respiratory rate are normal, and the newborn's cry is vigorous.

While the nurse is assessing the prenatal client's understanding of the information provided at a recent appointment, the client states, "I want to avoid the pain and long hours of labor, so I'm electing to have a cesarean birth." How should the nurse respond? "A cesarean birth is a method to be used when vaginal birth is not possible—it is not a true option." "It is important to inform the health care provider of your decision so that we can plan appropriately for when the birth occurs." "Cesarean birth is a low-risk surgery that is the ideal option in many cases." "Many women elect to have a cesarean birth to avoid the risk of anoxia."

"A cesarean birth is a method to be used when vaginal birth is not possible—it is not a true option." Explanation: It is important for the nurse to explain that cesarean birth is a method to be used when a vaginal birth is not possible and is not a true option. For this reason, the nurse will not inform the health care provider of the client's decision to plan for cesarean birth, although additional follow-up with the client's health care provider may be required. Cesarean birth is one of the safest types of surgeries, but the nurse should not present a cesarean birth as an ideal option in the absence of other supporting information. It is not the responsibility of the client to make the decision to undergo a cesarean birth to avoid the risk of anoxia.

A 35-year-old client is seen for her 2-week postoperative appointment after a suction curettage was performed to evacuate a hydatidiform mole. The nurse explains that the human chorionic gonadotropin (hCG) levels will be reviewed every 2 weeks and teaches about the need for reliable contraception for the next 6 months to a year. The client states, "I'm 35 already. Why do I have to wait that long to get pregnant again?" What is the nurse's best response? "After a curettage procedure, it is recommended that you give your body some time to build up its stores." "A contraceptive is used so that a positive pregnancy test resulting from a new pregnancy will not be confused with the increased level of hCG that occurs with a developing malignancy." "Since you are at the end of your reproductive years, it is suggested that you don't try to have any more pregnancies." "You may need chemotherapy, so we don't want to risk pregnancy."

"A contraceptive is used so that a positive pregnancy test resulting from a new pregnancy will not be confused with the increased level of hCG that occurs with a developing malignancy." Explanation: Because of the risk of choriocarcinoma, the woman receives extensive treatment. Therapy includes baseline chest X-ray to detect lung metastasis, plus a physical exam (including a pelvic exam). Serum B-hCG levels weekly until negative results are obtained three consecutive times, then monthly for 6 to 12 months. The woman is cautioned to avoid pregnancy during this time because the increasing B-hCG levels associated with pregnancy would cause confusion as to whether cancer had developed. If after a year B-hCG serum titers are within normal levels, a normal pregnancy can be achieved.

A 33-year-old multipara is 6 weeks' pregnant. The client states having always felt nauseous in the early part of pregnancy and is concerned about dietary intake at the beginning of this pregnancy because it seems the nauseous feeling is beginning earlier. How will the nurse counsel this client? "Nausea peaks at 6 weeks. Symptoms are probably at their peak." "As long as you do not develop any vomiting, your electrolytes will be maintained." "Alter your breakfast to light foods and have small, frequent meals throughout the day." "If you continue to feel nauseous, eat one full meal per day and sip on water as tolerated."

"Alter your breakfast to light foods and have small, frequent meals throughout the day." Explanation: Nausea peaks at 9 weeks and usually disappears by 14 weeks. A pregnant client should try to always keep something small in the stomach, opposed to a full meal, to avoid hypoglycemia which can exacerbate the nausea. Small or light and frequent meals are encouraged as they are most often able to be tolerated. Although it is good that the client is not vomiting, nausea can effect nutritional status, which is as much of a concern as electrolyte imbalance.

A pregnant client is experiencing a decreased nutritional status related to pyrosis. Which statement of advice by the nurse is best? "It is important for you to continue to eat three meals every day." "You can take as many antacids as you want since it also adds calcium to the diet." "Avoid lying down for at least 2 hours after you eat your meals." "Eat whenever you are hungry for you to maintain your intake and glucose level."

"Avoid lying down for at least 2 hours after you eat your meals." Explanation: Pyrosis, or heartburn, is a common discomfort of pregnancy. For clients with heartburn, it's best to eat smaller, more frequent meals and to avoid lying down for at least 2 hours to allow for gastric emptying. While it is safe to administer antacids throughout pregnancy, the safe dose is between 100 mg-1300 mg/day. Exceeding this dose may lead to side effects related to bicarbonate levels or elevated calcium levels. It is important to maintain a nutritious diet—not simply foods that the client likes—for the benefit of the mother and fetus.

The nurse cares for a client scheduled for a cesarean birth for a breech presentation. The client requests general anesthesia instead of regional anesthesia for the procedure because of increased anxiety about being awake during the surgery. How does the nurse respond? "Can you tell me more about what you are concerned about during the surgery?" "The general anesthetic creates a higher risk for your baby, so it is not recommended." "There will be a drape placed, so you do not have to worry about seeing the surgery." "Would you like to have your significant other present for the birth?"

"Can you tell me more about what you are concerned about during the surgery?" open ended helps problem solve

The nurse is conducting a teaching session for breastfeeding mothers. Which statement by a mother requires further clarification by the nurse? "I am glad I can have my two cups of coffee in the morning again." "I will continue to take a prenatal multivitamin as long as I am breastfeeding." "I will continue to add about 300 calories per day to my diet." "I will drink a large glass of water each time I nurse my baby."

"I am glad I can have my two cups of coffee in the morning again." Explanation: Breastfeeding mothers should avoid caffeine because it delays iron absorption and passes through the milk and can slow infant weight gain. Similarly, spicy foods pass into the breastmilk and can affect the baby. Breastfeeding mothers need added calories and fluids.

The nurse is administering a postpartum woman an antibiotic for mastitis. Which statement by the mother indicates that she understood the nurse's explanation of care? "I will stop breastfeeding until I finish my antibiotics." "I am able to pump my breast milk for my baby and throw away the milk." "I can continue breastfeeding my infant, but it may be somewhat uncomfortable." "When breastfeeding, it is recommended to begin nursing on the infected breast first."

"I can continue breastfeeding my infant, but it may be somewhat uncomfortable." Explanation: Breastfeeding on antibiotics for mastitis is fine, and the mother is encouraged to empty the infected breast to prevent milk stasis. However, the nurse should prepare the mother for the process being somewhat painful because the breast is tender. It is recommended to start the infant nursing on the uninvolved breast first as vigorous sucking may increase the mother's pain. Unless contraindicated by the antibiotic, the breast milk will be stored for later if the mother needs to pump her breasts; she does not need to throw the milk away.

A mother is talking to the nurse and is concerned about managing her asthma while she is pregnant. Which response to the nurse's teaching indicates that the woman needs further instruction? "I need to be aware of my triggers and avoid them as much as possible." "It is fine for me to use my albuterol inhaler if I begin to feel tight." "I need to begin taking allergy shots like my friend to prevent me from having an allergic reaction this spring." "I will monitor my peak expiratory flow rate regularly to help me predict when an asthma attack is coming on."

"I need to begin taking allergy shots like my friend to prevent me from having an allergic reaction this spring." Explanation: A pregnant woman with a history of asthma needs to be proactive, taking her inhalers and other asthma medications to prevent an acute asthma attack. She needs to understand that it is far more dangerous to not take the medications and have an asthma attack. She also needs to monitor her peak flow for decreases, be aware of triggers, and avoid them if possible. However, a pregnant woman should never begin allergy shots if she has not been taking them previously, due to the potential of an adverse reaction.

A pregnant woman with chronic hypertension is entering her second trimester. The nurse is providing anticipatory guidance to the woman about measures to promote a healthy outcome. The nurse determines that the teaching was successful based on which client statement(s)? Select all that apply. "I will need to schedule follow-up appointments every 2 weeks until I reach 32 weeks' gestation." "I should try to lie down and rest on my left side for about an hour each day." "I will start doing daily counts of my baby's activity at about 24 weeks' gestation." "I will need to have an ultrasound at each visit beginning at 28 weeks' gestation." "I should take my blood pressure frequently at home and report any high readings."

"I should try to lie down and rest on my left side for about an hour each day." "I will start doing daily counts of my baby's activity at about 24 weeks' gestation." "I should take my blood pressure frequently at home and report any high readings." The woman with chronic hypertension will be seen more frequently (every 2 weeks until 28 weeks' gestation and then weekly until birth) to monitor her blood pressure and to assess for any signs of preeclampsia. At approximately 24 weeks' gestation, the woman will be instructed to document fetal movement. At this same time, serial ultrasounds will be prescribed to monitor fetal growth and amniotic fluid volume. The woman should also have daily periods of rest (1 hour) in the left lateral recumbent position to maximize placental perfusion and use home blood pressure monitoring devices frequently (daily checks would be preferred), reporting any elevations.

A pregnant client with a history of asthma since childhood presents for a prenatal visit. What statement by the client alerts the nurse to perform further assessment? "I sometimes get a bit wheezy." "I have trouble getting comfortable in bed." "I sometimes get a feeling of euphoria." "Certain substances make me sneeze."

"I sometimes get a bit wheezy." Explanation: Wheezing is a classic symptom of asthma. This statement should alert the nurse to the possibility that the client's asthma is not being well-controlled and needs further evaluation and possible intervention. The other statements do not relate to the typical presentation of this disease in pregnancy nor are they cause for concern.

The nurse is teaching the pregnant woman about nutrition for herself and her baby. Which statement by the woman indicates that the teaching was effective? "I can eat any seafood that I like because it contains phosphorus, which is a nutrient that pregnant women need." "I will need to take iron supplementation throughout my pregnancy even if I am not anemic." "Milk production requires higher levels of calcium; therefore, if I am going to breastfeed, I must take a calcium supplement during pregnancy." "Because I am pregnant, I can eat anything I want and not worry about weight gain."

"I will need to take iron supplementation throughout my pregnancy even if I am not anemic." Explanation: Iron is recommended for all pregnant women because it is almost impossible for the pregnant woman to get what is required from diet alone, especially after 20 weeks' gestation when the requirements of the fetus increase. Pregnant women can get many nutrients from seafood including phosphorus, but there are specific recommendations about types of fish to avoid because of the risk of mercury poisoning. Milk production actually requires higher levels of zinc, which can be obtained from a healthy diet. Calcium requirements do not increase above prepregnancy levels during pregnancy because calcium absorption is enhanced during pregnancy. It can be unsafe for the pregnant woman to eat anything she wants and gain too much weight. A woman who gains too much weight during pregnancy is at risk for delivering a macrosomic baby.

A 32-year-old woman with epilepsy mentions to the nurse during a routine well-visit that she would like to have children and asks the nurse for advice. Which response is most appropriate from the nurse? "You should talk to the doctor about that; the medications you're on can damage the fetus." "Do you want to talk to a counselor who can help you weigh the pros and cons of having your own child rather than adopting?" "I'll let the doctor know so you can discuss your medications. In the meantime, I'll give you a list of folate-rich foods you can add to your diet." "That's great. I've got a 4-year-old and a 2-year-old myself."

"I'll let the doctor know so you can discuss your medications. In the meantime, I'll give you a list of folate-rich foods you can add to your diet." Explanation: Any woman with epilepsy needs to discuss medication management with her provider. The current research indicates the medications used for epileptic management are the major cause of birth defects for these clients. The nurse should be careful about mentioning that some epilepsy medications are teratogenic; some women may stop taking their medications in order to get pregnant. Suggesting adoption is inappropriate as the mother has given no indication she is interested in adoption; also, the mother needs to discuss this with the physician so that she can get accurate information about being on anti-seizure medications and being pregnant. The nurse should not share personal information as it does not assist this client in making a serious decision. The client should be referred to the health care provider to help the client make the best decision.

The nurse is teaching a client with gestational diabetes about complications that can occur either following birth or during the birth for the infant. Which statement by the mother indicates that further teaching is needed by the nurse? "My baby may be very large and I may need a cesarean birth to have him." "If my blood sugars are elevated, my baby's lungs will mature faster, which is good." "Beginning at 28 weeks' gestation, I will start counting with my baby's movements every day." "I may need an amniocentesis during the third trimester to see if my baby's lungs are ready to be born."

"If my blood sugars are elevated, my baby's lungs will mature faster, which is good." Explanation: Elevated blood sugars delay the maturation of fetal lungs, not increase maturation time, resulting in potential respiratory distress in newborns born to mothers with diabetes. Doing fetal movement (kick) counts is standard practice, as is the possibility of an amniocentesis to determine lung maturity during the third trimester. Health care personnel should also prepare the mother for the potential of a cesarean birth if the infant is too large.

The pinna of the ear is soft and flat and stays folded. Explanation: The ear has a soft pinna that is flat and stays folded. Pale skin with no vessels showing through and 7 to 10 mm of breast tissue are characteristic of a neonate at 40 weeks' gestation. Creases on the anterior two-thirds of the sole are characteristic of a neonate at 36 weeks' gestation.

"It is a normal skin finding in a newborn." Explanation: This most likely is erythema toxicum, also known as newborn rash, and is a common finding that will gradually disappear and not need any treatment. This is often mistaken for staphylococcal pustules. This is not a sign of mistreatment by the woman, nor is it caused by a virus or group B streptococcal (GBS) infection.

A pregnant women calls the clinic to report a small amount of painless vaginal bleeding. What response by the nurse is best? "Please come in now for an evaluation by your health care provider." "Lie on your left side and drink lots of water and monitor the bleeding." "If the bleeding lasts more than 24 hours, call us for an appointment." "Bleeding during pregnancy happens for many reasons, some serious and some harmless."

"Please come in now for an evaluation by your health care provider." Explanation: Bleeding during pregnancy is always a deviation from normal and should be evaluated carefully. It may be life-threatening or it may be something that is not a threat to the mother and/or fetus. Regardless, it needs to be evaluated quickly and carefully. Telling the client it may be harmless is a reassuring statement, but does not suggest the need for urgent evaluation. Having the mother lay on her left side and drink water is indicated for cramping.

A client with a history of cervical insufficiency is seen for reports of pink-tinged discharge and pelvic pressure. The primary care provider decides to perform a cervical cerclage. The nurse teaches the client about the procedure. Which client response indicates that the teaching has been effective? "Staples are put in the cervix to prevent it from dilating." "The cervix is glued shut so no amniotic fluid can escape." "Purse-string sutures are placed in the cervix to prevent it from dilating." "A cervical cap is placed so no amniotic fluid can escape."

"Purse-string sutures are placed in the cervix to prevent it from dilating." Explanation: The cerclage, or purse string suture, is inserted into the cervix to prevent preterm cervical dilation (dilatation) and pregnancy loss. Staples, glue, or a cervical cap will not prevent the cervix from dilating.

When collecting data to devise a labor plan for a multiparous woman, which question best allows the nurse to develop individualized strategies? "Tell me how you handled labor pain in your past deliveries." "How do you want the health care team to plan your care?" "Who do you want to be with you when you are in labor?" "Picking from these options, what options do you feel is best?"

"Tell me how you handled labor pain in your past deliveries." Explanation: When the nurse is collecting data, it is best to discuss previous experiences with labor pain. Other questions may include, "What was helpful?" or "What did you not like?" While it is true that every labor is different, understanding the client's perspective from past experiences is valuable in developing individualized strategies. Developing a plan is best as a collaborative effort, not by picking pre-prepared options. It is important to include a support person if desired.

A woman in labor has chosen to use hydrotherapy as a method of pain relief. Which statement by the woman would lead the nurse to suspect that the woman needs additional teaching? "The warmth and buoyancy of the water has a nice relaxing effect." "I can stay in the bath for as long as I feel comfortable." "My cervix should be dilated more than 5 cm before I try using this method." "The temperature of the water should be at least 105℉ (40.5℃)."

"The temperature of the water should be at least 105℉ (40.5℃)." Explanation: Hydrotherapy is an effective pain relief method. The water temperature should not exceed body temperature. Therefore, a temperature of 105℉ (40.5℃) would be too warm. The warmth and buoyancy have a relaxing effect, and women are encouraged to stay in the bath as long as they feel comfortable. The woman should be in active labor with cervical dilation greater than 5 cm.

A nurse is teaching a woman in her third trimester about Braxton Hicks contractions. When describing these contractions, which information would the nurse likely include? Select all that apply. "They usually feel like a tightening across the top of your uterus." "They typically last for about 3 minutes each time you have them." "They often spread downward before they go away." "They usually happen in a regular pattern." "They go away when you walk around or change position."

"They go away when you walk around or change position." "They often spread downward before they go away." "They usually feel like a tightening across the top of your uterus." Braxton Hicks contractions are typically felt as a tightening or pulling sensation of the top of the uterus. They occur primarily in the abdomen and groin and gradually spread downward before relaxing. In contrast, true labor contractions are more commonly felt in the lower back. These contractions aid in moving the cervix from a posterior position to an anterior position. They also help in ripening and softening the cervix. However, the contractions are irregular and can be decreased by walking, voiding, eating, increasing fluid intake, or changing position. Braxton Hicks contractions usually last about 30 seconds but can persist for as long as 2 minutes. As birth draws near and the uterus becomes more sensitive to oxytocin, the frequency and intensity of these contractions increase. However, if the contractions last longer than 30 seconds and occur more often than four to six times an hour, the woman should be advised to contact her health care provider to be evaluated, especially if she is less than 39 weeks' pregnant.

A parent asks the nurse how to swaddle the newborn because the parent heard that it helps newborns calm down. Which statement will the nurse include in the teaching? "Newborns swaddled frequently may not respond to this comfort measure." "It is best if you use the same blanket each time for swaddling." "Wrapping the newborn too tightly can impair breathing." "The newborn needs to be held after being swaddled."

"Wrapping the newborn too tightly can impair breathing." Explanation: Swaddling is a useful measure to comfort a fretful newborn. The only identified problem is that the newborn can become too tightly wrapped, leading to respiratory compromise and breathing difficulties. Swaddling reduces the need to be held, there is no risk of the newborn not responding to it after being swaddled in the past, and the parent does not have to use the same blanket every time.

A client is 12 weeks' pregnant and asks the nurse about obtaining dental care as she just obtained dental insurance through a new job. Which nursing instruction regarding dental care is an appropriate response? "You should schedule a dental appointment with your dentist now for preventive care, but avoid any nonemergent dental work until after the baby is born." "The dentist will obtain a baseline of your oral status. As long as a protective covering is worn, dental x-rays are completely safe in pregnancy." "Oral health is important, but as long as you brush twice a day, floss regularly, and have no pain, you can wait until after you have the baby to see the dentist." "The dentist can make a basic treatment plan to manage your gingivitis following birth. Gingivitis should be treated only after the baby is born."

"You should schedule a dental appointment with your dentist now for preventive care, but avoid any nonemergent dental work until after the baby is born." Explanation: After being away from routine dental care, the nurse is most correct to encourage dental consultation for this client. The client should continue regular dental care while pregnant but unnecessary dental work should be avoided. Untreated gingivitis can damage the gums, resulting in bone and tooth loss. Treatment is necessary by weighing the risks and benefits. X-rays are not recommended in the first trimester due to the theoretical risk of radiation exposure to the fetus during the embryonic stage. Good oral hygiene includes brushing, flossing, and routine dental visits.

A primigravida at her 12-week prenatal visit expresses concern that she hasn't felt her baby move yet. What is the best response from the nurse? "Fetal movements can be felt at 13 weeks." "You should start to feel fetal movements within the next few weeks." "You usually cannot feel them until approximately 16 to 20 weeks." "You won't be able to feel movements until you lie down and concentrate on them."

"You usually cannot feel them until approximately 16 to 20 weeks." Explanation: The first fetal movements felt by the pregnant woman are usually felt between 16 and 20 weeks gestation. Thirteen weeks is too early (fetus is too small), and movements cannot be felt even if the woman lies down and concentrates on them, although the woman should start feeling movements within the next few weeks.

A woman is admitted to the labor and birthing suite. Vaginal examination reveals that the presenting part is approximately 2 cm above the ischial spines. The nurse documents this finding as: +2 station. 0 station. -2 station. crowning.

-2 station. Explanation: The ischial spines serve as landmarks and are designated as zero status. If the presenting part is palpated higher than the maternal ischial spines, a negative number is assigned. Therefore, the nurse would document the finding as -2 station. If the presenting part is below the ischial spines, then the station would be +2. Crowning refers to the appearance of the fetal head at the vaginal opening.

A urinalysis is done on a client in her third trimester. Which result would be considered abnormal? Trace of glucose 2+ Protein in urine Specific gravity of 1.010 Straw-like color

2+ Protein in urine Explanation: During pregnancy, there may be a slight amount of glucose found in the urine due to the fact that the kidney tubules are not able to absorb as much glucose as there were before pregnancy. However, there should be minimal protein in the urine. A specific gravity of 1.010 and a straw- like color are both normal findings.

How long is the neonatal period for a newborn? 28 days 14 days 90 days 45 days

28 days Explanation: The neonatal period is the first 28 days of life.

A client who is 4 months' pregnant is at the prenatal clinic for the initial visit. The history reveals the client has 7-year-old twins who were born at 34 weeks' gestation, a 2-year-old child born at 39 weeks' gestation, and a spontaneous abortion (miscarriage) 1 year ago at 6 weeks' gestation. Using the GTPAL method, the nurse would document the obstetric history as:

4 1 1 1 3. Explanation: Using the GTPAL method, the client's history would be documented as 4 (the fourth pregnancy), 1 (number of term pregnancies), 1 (number of pregnancies ending in preterm birth), 1 (number of pregnancies ending before 20 weeks or viability), and 3 (number of living children).

A woman at 38 weeks' gestation arrives at the hospital reporting a sharp pain between her umbilicus and the iliac crest that is increasing in intensity in the right lower quadrant. She reports having experienced intense nausea and vomiting for the past 3 hours. The nurse will report which recommendation to the primary health care provider? A diagnostic test such as an abdominal ultrasound A fetal nonstress test and fetal movement (kick) counts An abdominal assessment with bowel sounds A urine culture and broad-spectrum antibiotic

A diagnostic test such as an abdominal ultrasound Explanation: All of the symptoms point to a diagnosis of acute appendicitis. With appendicitis, the nausea and vomiting are much more intense than with morning sickness and the pain is sharp and localized at McBurney's point (a point halfway between the umbilicus and the iliac crest on the lower right abdomen). When identifying, it is best to recommend a diagnostic test such as an abdominal ultrasound to confirm the diagnosis. It is important to confirm the diagnosis as surgery needs to be completed to prevent a rupture. It is important to do an abdominal assessment but that will not confirm a diagnosis. The symptoms do not indicate a urinary issue. There is no data to indicate an issue with the fetus.

Hormone levels of a woman indicate that the corpus luteum stopped functioning and releasing progesterone after 5 weeks. The nurse would recognize that which scenario is the expected outcome? The pregnancy would continue unaffected. A spontaneous abortion (miscarriage) would occur. There is a higher than normal chance of a multifetal pregnancy. She will need progesterone supplement throughout the pregnancy.

A spontaneous abortion (miscarriage) would occur. Explanation: If the corpus luteum fails to produce progesterone for approximately 6 to 7 weeks, a spontaneous abortion (miscarriage) will occur. After 7 weeks, the placenta will produce enough progesterone to sustain the pregnancy. There is no connection between multifetal pregnancies and the corpus luteum not functioning long enough in progesterone production.

How should the nurse counsel a client who has arrived for a scheduled repeat cesarean birth? "You will undergo general anesthesia, so you will be asleep during the procedure." "A Foley catheter will be inserted before surgery and will be removed once you arrive to the postpartum unit." "After surgery, you will be immediately transferred to the postpartum unit." "An IV catheter will be placed, and we will do some preoperative blood work. Then we will give you some antibiotics."

An IV catheter will be placed, and we will do some preoperative blood work. Then we will give you some antibiotics." Explanation: Epidural or spinal anesthesia is most commonly used for a scheduled cesarean, and the woman is awake during the procedure. Immediately after the procedure, the woman is transferred to the postanesthesia care unit or the recovery unit to recover from the surgery for 1 to 2 hours. The Foley usually remains in place for 24 hours.

Which statement regarding newborn circumcision is accurate? An advantage of circumcision is a decreased risk of penile cancer. A disadvantage of circumcision is a higher risk of sexually transmitted infections. The American Academy of Pediatrics (AAP) currently discourages circumcision. Newborns do not experience pain during a circumcision.

An advantage of circumcision is a decreased risk of penile cancer. Explanation: Advantages of newborn circumcision are decreased risk of penile cancer and decreased risk of sexually transmitted infections. The AAP states that the health benefits outweigh the risks of newborn circumcision. Research has shown that newborns do experience pain with circumcision.

A woman in active labor suddenly sits up, clutches her chest, screams with pain, and then collapses back on the bed. The RN notes she is unconscious and a bluish-gray color. Which interventions are considered the priority for the nurse to implement? Select all that apply. Apply oxygen mask and start oxygen at 10 L/min. Begin CPR immediately. Place stethoscope on the abdomen to verify fetal heart rate. Start oxytocin at 4 mu/min and titrate upward every 5 minutes. Call lab and request 4 units of whole blood stat.

Apply oxygen mask and start oxygen at 10 L/min. Begin CPR immediately. Amniotic fluid embolism occurs when amniotic fluid is forced into an open maternal uterine blood sinus after a membrane rupture or partial premature separation of the placenta. The clinical picture is dramatic. The immediate management is oxygen administration by face mask or cannula. Within minutes, she will need CPR; however, CPR may be ineffective because these procedures do not relieve the pulmonary constriction. Blood still cannot circulate to the lungs. Death may occur within minutes. Taking time to listen to FHR is not the priority. Oxytocin will not help with embolism. If the woman survives and develops DIC, fibrinogen is the blood product of choice.

A woman in scrubs enters a mother's room while the nurse is completing an assessment. The woman states the doctor is in the nursery and has requested the infant be brought back for an examination. What will the nurse do? Ask the woman to bring the infant back when the doctor finishes the examination. Call the nursery to confirm the doctor does indeed need this infant at this time. Ask to see the woman' hospital identification badge. Ask how long the infant will be gone since her next feeding is in 30 minutes.

Ask to see the woman' hospital identification badge. Explanation: The nurse will not release an infant to anyone who does not have a hospital photo ID that matches the security color or code for the hospital, indicating that they are authorized to transport infants. Asking the woman to bring the newborn back, calling the nursery, or determining how long the newborn will be gone do not address the security issue.

The nurse is reviewing the medication administration record (MAR) of a client at 39 weeks' gestation and notes that she is ordered an opioid for pain relief. Which is an assessment priority after administering? Assess maternal blood pressure. Assess for constipation. Assess for dry mouth. Assess fetal heart rate.

Assess fetal heart rate. Explanation: After administering an opioid to a laboring mother, the priority is to assess the impact on the fetus. Opioid administration can cross the placental barrier and affect fetal heart rate and variability. After birth, there may be a decrease in alertness of the neonate. Maternal factors of decreased blood pressure, constipation, and dry mouth are of a lower priority.

A client's membranes have just ruptured. Her fetus is presenting breech. Which action should the nurse do immediately to rule out prolapse of the umbilical cord in this client? Assess fetal heart sounds. Place the woman in Trendelenburg position. Administer oxygen at 10 L/min by face mask. Administer amnioinfusion.

Assess fetal heart sounds. Explanation: To rule out cord prolapse, always assess fetal heart sounds immediately after rupture of the membranes whether this occurs spontaneously or by amniotomy, as the fetal heart rate will be unusually slow or a variable deceleration pattern will become apparent when cord prolapse has occurred. The other answers refer to therapeutic interventions to implement once cord prolapse has been confirmed.

The nurse observes an ambulating postpartum woman limping and avoiding putting pressure on her right leg. Which assessments should the nurse prioritize in this client? Bend the knee and palpate the calf for pain. Ask the client to raise the foot and draw a circle. Blanch a toe, and count the seconds it takes to color again. Assess for warmth, erythema, and pedal edema.

Assess for warmth, erythema, and pedal edema. Explanation: This client is demonstrating potential symptoms of DVT, but is avoiding putting pressure on the leg and limping when ambulating. DVT manifestations are caused by inflammation and obstruction of venous return and can be assessed by the presence of calf swelling, warmth, erythema, tenderness, and pedal edema. The client would not need to bend the knee to assess for pain in the calf. Asking the client to raise her toe and draw a circle is assessing reflexes, and blanching a toe is assessing capillary refill (which may be affected by the DVT but is not indicative of a DVT)

The nurse has completed an assessment on a 1-day-old newborn. Which finding should the nurse prioritize? temperature of 97.8°F (36.5°C) heart rate 158 bpm respiratory rate 42 breaths/min Blood sugar 42 mg/dl

Blood sugar 42 mg/dl Explanation: Any blood sugar lower than 50 mg/dl is considered hypoglycemic and should be further assessed. In the scenario described, the infant's temperature, heart rate, and respiratory rate are all considered within normal limits.

x A nurse is called into the room of one of the clients where the grandparents are visiting. The grandmother is visibly upset, and says "Just look at my grandson! His head is all soft and swollen here and it shouldn't be. The doctor injured him when he was born." The nurse assesses the newborn and finds an area of swelling about the size of a half-dollar at the center of the upper scalp. The nurse determines this finding is most likely which condition? Increased intracranial pressure Caput succedaneum Molding Harlequin sign

Caput succedaneum Explanation: Caput succedaneum is swelling of the soft tissue of the scalp caused by pressure of the presenting part on a partially dilated cervix or trauma from a vacuum-assisted delivery. This finding is often of concern for the families. Reassure them that the caput will decrease in a few days without treatment. Increased intracranial pressure would involve the entire scalp and not just a small portion. There would also be other neurologic signs accompanying it. Molding is an elongated head shape caused by overlapping of the cranial bones as the fetus moves through the birth canal. This will also resolve in a few days without treatment. The Harlequin sign is characterized by a clown-suit-like appearance of the newborn where the skin is dark red on one side of the body and the other side is pale. This is a harmless condition which occurs most frequently with vigorous crying or with the infant lying on his or her side.

The nurse is comforting and listening to a young couple who just suffered a spontaneous abortion (miscarriage). When asked why this happened, which reason should the nurse share as a common cause? Maternal smoking Lack of prenatal care Chromosomal abnormality The age of the mother

Chromosomal abnormality Explanation: The most common cause for the loss of a fetus in the first trimester is associated with a genetic defect or chromosomal abnormality. There is nothing that can be done and the mother should feel no fault. The nurse needs to encourage the parents to speak with a health care provider for further information and questions related to genetic testing. Early pregnancy loss is not associated with maternal smoking, lack of prenatal care, or the age of the mother.

The nurse is documenting the length of time in the second stage of labor. Which data will the nurse use to complete the documentation? Admission time and time of fetal birth Complete cervical dilation (dilatation) and time of fetal birth Effacement time and time when contractions are regular Time of mucus plug expulsion and full cervical dilation

Complete cervical dilation (dilatation) and time of fetal birth Explanation: The second stage of labor begins with complete cervical dilation (dilatation) of 10 cm and ends with delivery of the neonate.

The nurse cares for a pregnant client in labor and determines the fetus is in the right occiput anterior (ROA) position. Which action by the nurse is best? Continue to monitor the progress of labor. Auscultate fetal heart rate (FHR) in the left upper quadrant. Prepare the client for cesarean birth of the fetus. Educate the client this fetal position may result in a longer labor.

Continue to monitor the progress of labor. Explanation: ROA (right occiput anterior) means the occiput of the fetal head points toward the mother's right anterior pelvis; the head is the presenting part. This is a common and favorable position for vaginal birth. Based on the ROA location, the nurse will auscultate FHR at the right lower quadrant of the client's abdomen (FHR will be loudest where the fetus' back is located). Occiput posterior (not anterior) positions are associated with longer, more difficult births.

During a postpartum parenting class, a client reports that she has switched her 6-month-old from formula to cow's milk to save money. Which of the following responses from the nurse would be most appropriate? "Cow's milk has less protein than formula, so your infant will take longer to recover from illness." "Cow's milk has more iron than formula, so the child may experience constipation." "Cow's milk should not be used with infants younger than 6 months." "Cow's milk should not be used with infants younger than 9 months."

Cow's milk should not be used with infants younger than 9 months." Explanation: The quality and quantity of nutrients in cow's milk differ greatly from those of formula. Cow's milk contains more protein and minerals than formula but less iron, which is poorly absorbed. To lower the risk of iron-deficiency anemia, cow's milk is not recommended for infants younger than 9 months.

Which cardinal movement of delivery is the nurse correct to document by station? Descent Flexion Extension Internal rotation

Descent Explanation: Descent is documented by station, which is the relationship of the fetal presenting part to the maternal ischial spines. Descent continues throughout labor until the fetus reaches the fetal station of +4. The other options represent fetal movements to accommodate the passage of the fetus.

Which primary symptom does the nurse identify as a potentially fatal complication of epidural or intrathecal anesthesia? Difficulty breathing Staggering gait Decreased level of consciousness Intense pain

Difficulty breathing Explanation: Total spinal blockade occurs when an inadvertent injection of a local anesthetic is placed into the intrathecal or epidural space. The resulting effect is that the anesthetic travels too high in the body causing paralysis of the respiratory muscles. Difficulty breathing is a sign. A decreased level of consciousness will occur later. A staggering gait or intense pain is not a primary symptom

The nurse is assessing a newborn's vital signs and notes the following: HR 138, RR 42, temperature 98.7oF (37.1oC), and blood pressure 70/40 mm Hg. Which action should the nurse prioritize? Report tachypnea. Recheck blood pressure in 15 minutes. Put warming blanket over infant. Document normal findings.

Document normal findings. Explanation: These vital signs are within normal limits and should be documented. The heart rate should be 110 to 160 bpm; RR should be 30 to 60 breaths per minute. The axillary temperature can range from 97.7°F to 99.6°F (36.5°C to 37.5°C). Blood pressure should be 60-80/40-45 mm Hg. There is no need to contact the health care provider, recheck the blood pressure in 15 minutes, or place a blanket on the infant.

A client in the first trimester reports having nausea and vomiting, especially in the morning. Which instruction would be most appropriate to help prevent or reduce the client's compliant? Drink plenty of fluids at bedtime. Avoid foods such as cheese. Avoid eating spicy food. Eat dry crackers or toast before rising.

Eat dry crackers or toast before rising. Explanation: The nurse should recommend the client eat dry crackers or toast before rising to prevent nausea and vomiting in the morning. Drinking plenty of fluids at bedtime could cause nocturia. Foods such as cheese should be avoided to prevent constipation. Spicy foods could cause heartburn.

A client in the second trimester of pregnancy reports heartburn. Which will the nurse suggest to the client as a preventive measure? Drink plenty of fluids before bedtime. Use extra pillows when sleeping. Eat small, frequent meals. Eat dry crackers and toast before rising.

Eat small, frequent meals. Explanation: The nurse will instruct the client to eat small, frequent meals to prevent heartburn. This helps the stomach remain full without getting overfilled, which increases the pressure in the stomach. It also helps to prevent excessive production of stomach acid. Drinking plenty of fluids before bedtime can cause nocturia. Use of extra pillows when sleeping is advised for clients to alleviate breathing difficulties. Eating dry crackers and toast before rising is helpful for clients with morning sickness.

A 28-year-old client with a history of endometriosis presents to the emergency department with severe abdominal pain and nausea and vomiting. The client also reports her periods are irregular with the last one being 2 months ago. The nurse prepares to assess for which possible cause for this client's complaints? Healthy pregnancy Ectopic pregnancy Molar pregnancy Placenta previa

Ectopic pregnancy Explanation: The most commonly reported symptoms of ectopic pregnancy are pelvic pain and/or vaginal spotting. Other symptoms of early pregnancy, such as breast tenderness, nausea, and vomiting, may also be present. The diagnosis is not always immediately apparent because many women present with complaints of diffuse abdominal pain and minimal to no vaginal bleeding. Steps are taken to diagnose the disorder and rule out other causes of abdominal pain. Given the history of the client and the amount of pain, the possibility of ectopic pregnancy needs to be considered. A healthy pregnancy would not present with severe abdominal pain unless the client were term and she was in labor. With a molar pregnancy the woman typically presents between 8 to 16 weeks' gestation reporting painless (usually) brown to bright red vaginal bleeding. Placenta previa typically presents with painless, bright red bleeding that begins with no warning.

Which assessment findings of the fetus during labor are normal? Select all that apply. Variability between 18-20 bpm Late decelerations Fetal heart baseline of 130 bpm Repeated variable decelerations Gradual increase in the fetal heart rate baseline

Fetal heart baseline of 130 bpm Variability between 18-20 bpm Explanation: Normal patterns suggest that the fetus is tolerating the labor. Both variability between 18-20 bpm and a baseline heart rate of 130 bpm are within normal limits. Both late and repeated variable decelerations are abnormal and may require further intervention. A gradual increase in the fetal heart rate baseline can signal a distressed fetus.

Which consideration is a priority when caring for a mother with strong contractions 1 minute apart? Fetal heart rate in relation to contractions The station in which the fetus is located Maternal heart rate and blood pressure Maternal request for pain medication

Fetal heart rate in relation to contractions Explanation: The priority consideration is on the status of the fetus. Because each contraction temporarily interrupts blood flow to the placenta, there is a decrease in oxygen available. Therefore, a fetus cannot tolerate contractions lasting too long or too strong. All other options are important but not the priority.

The nurse is admitting a client who is in early labor. After determining that the birth is not imminent, which assessment should the nurse perform next? Risk factors Maternal status Fetal status Maternal obstetrical history

Fetal status Explanation: The woman may present to the birthing suite at any phase of the first stage of labor. Therefore, it is important to assess birth imminence, fetal status, risk factors, and maternal status immediately. If birth is not imminent and the fetal and maternal conditions are stable, perform additional data collection, including the full admission health history, a complete maternal physical assessment, the status of labor and any labor, birth, and cultural preferences the woman may have.

Fetal circulation differs from the circulatory path of the newborn infant. In utero the fetus has a hole connecting the right and left atria of the heart. This allows oxygenated blood to quickly pass to the major organs of the body. What is this hole called? Foramen venosus Foramen magnum Foramen arteriosus Foramen ovale

Foramen ovale Explanation: The foramen ovale is a hole that connects the right and left atria so the majority of oxygenated blood can quickly pass into the left side of the fetal heart, go to the brain, and move to the rest of the fetal body.

A pregnant client at 18 weeks' gestation has arrived for her routine prenatal visit. Which assessment findings should the nurse prepare to document at this time? Select all that apply. Fundal height of approximately 18 cm Quickening Insomnia Braxton Hicks contractions Leg cramps

Fundal height of approximately 18 cm Quickening Between 18 and 30 weeks' gestation, fundal height in centimeters is approximately the same as the number of weeks' gestation. In this case, the client is 18 weeks pregnant, so fundal height should measure approximately 18 cm. Quickening, which is typically described as light fluttering and is usually felt between 16 and 22 weeks' gestation, is caused by fetal movement. Insomnia, Braxton Hicks contractions, and leg cramps are common during the third trimester.

The nurse is monitoring the electronic fetal heart rate monitor and notes the following: variable V-shaped decelerations in the fetal heart rate (FH)R lasting about 30 seconds, accelerations of about 5 beats/min before and after each deceleration, no overshoot, and baseline FHR within normal limits. Which response should the nurse prioritize? Help the woman change positions. Discontinue supplemental oxygen. Encourage pushing with contractions during second stage of labor. Start an oxytocic infusion and decrease the rate of IV fluids.

Help the woman change positions. Explanation: The electronic fetal heart rate monitor reading is associated with cord compression. Changing to a different position is a first intervention to determine if this will improve the oxygen to the fetus. Supplemental oxygen should be maintained until the mother is stable. Pharmacologic interventions are premature. The nurse should modify pushing in the second stage of labor to improve fetal oxygenation.

A student nurse is preparing for a presentation that will illustrate the various physiologic changes in the woman's body during pregnancy. Which cardiovascular changes up through the 26th week should the student point out? Decreased pulse rate and increased blood pressure Increased pulse rate and decreased blood pressure Increased pulse rate and blood pressure No change in pulse rate or blood pressure

Increased pulse rate and decreased blood pressure Explanation: Pulse rate frequently increases during pregnancy, although the amount varies from a slight increase to 10 to 15 beats per minute. Blood pressure generally decreases slightly during pregnancy, reaching its lowest point during the second trimester.

Which is true regarding mineral requirements in the newborn? Infants who are breastfed need supplemental iron. Tetany from inadequate calcium intake is likely to occur in a breastfed infant, not in a formula-fed infant. Mothers who are breastfeeding should drink spring water only. Infants who are formula-fed should drink an iron-enriched formula for at least 12 months.

Infants who are formula-fed should drink an iron-enriched formula for at least 12 months. Explanation: Breast milk contains an adequate amount of iron, and infants who are breastfed do not need to supplement with oral iron. However, infants do not make adult hemoglobin until 3 to 6 months of age; they therefore need iron-enriched formula. The American Academy of Pediatrics recommends iron-enriched formula for at least 1 year. Mothers who are breastfeeding should drink some fluoridated water. Spring water is not enriched with fluoride.

The nurse is preparing a newborn male for circumcision. During the assessment, the nurse notes the newborn has a hypospadias. Which action made by the nurse is best? Continue to prepare the newborn for the procedure. Tell the parents the procedure may take more time because of the hypospadias. Give the newborn a sucrose pacifier to reduce pain during the procedure. Inform the practitioner and cancel the procedure.

Inform the practitioner and cancel the procedure. Explanation: Hypospadias is contraindicated for circumcision. The circumcision is postponed so that the foreskin can be used to repair the hypospadias. Sucrose water can be given for pain relief in circumcision; however, it would not be given if the procedure is cancelled.

A newborn infant at 36 hours of age is jaundiced. The mother is breastfeeding. What intervention is appropriate to increase the excretion of bilirubin? Stop breastfeeding and administer glucose water for 24 hours. Instruct the mom to feed every two to three hours. Restrict feedings and give glucose water every 4-6 hours for hydration. Keep the skin protected by preventing light onto the baby's skin.

Instruct the mom to feed every two to three hours. Explanation: Bilirubin is excreted in the urine and feces. Encouraging the mother to breastfeed at least every two to three hours will increase the waste and help decrease the bilirubin level. Stopping breastfeeding and administering glucose water for 24 hours would not be appropriate for the mother. Restricting feedings and giving glucose water every 4 to 6 hours is not an appropriate nursing intervention for an infant showing signs of jaundice. Keeping light away from the baby's skin does not help to clear jaundice; it could only make it worse.

The nursing instructor is teaching a class on the physiologic prosperities involved with the birthing process. The instructor determines the session is successful when the students correctly match surfactant with which function? It expands the lungs with breaths. It keeps alveoli from collapsing with breaths. It removes fluid from the lungs. It allows oxygen to move in the lungs.

It keeps alveoli from collapsing with breaths. Explanation: The role of surfactant is to act on surface tension and assist in keeping the alveoli open in the lungs so the lungs do not collapse with the respiratory effort of the newborn. Surfactant does not expand the lungs, remove fluid from the lungs, or allow oxygen to move in the lungs.

The nurse is praising an adolescent for seeking health care as soon as the adolescent found out about being pregnant. Which nursing intervention is the priority for this client in the first trimester of pregnancy? Schedule the client for a screening glucose tolerance test. Make sure the client receives nutritional counseling and reinforce the teaching. Teach the client about needing 8 to 10 hours of sleep each night. Instruct on fetal development throughout the pregnancy.

Make sure the client receives nutritional counseling and reinforce the teaching. Explanation: There are many important nursing interventions for an adolescent who is pregnant. Nutritional counseling must be emphasized as part of prenatal care for adolescent clients because adolescents already have higher nutritional demands due to their growth status. Nutrition is also a priority due to the fetus' development. Adolescents are not at increased risk for developing gestational diabetes, so the client does not need a glucose tolerance test at this time. Adolescents do need 8 to 10 hours of sleep per night, but this is not the priority education over nutrition education. Instruction on fetal development at the first visit may be overwhelming and is not the priority at this time.

General anesthesia is not used frequently in obstetrics because of the risks involved. There are physiologic changes that occur during pregnancy that make the risks of general anesthesia higher than it is in the general population. What is one of those risks? The client is more sensitive to preanesthetic medications. The client is less sensitive to inhalation anesthetics. Neonatal depression is possible. Fetal hypersensitivity to anesthetic is possible.

Neonatal depression is possible. Explanation: General anesthesia is not used frequently in obstetrics because of the risks involved. The pregnant woman is at higher risk for aspiration. It requires more skill to intubate a pregnant woman because of physiologic changes in the trachea and thorax. In addition, general anesthetic agents cross the placenta and can result in the birth of a severely depressed neonate who requires full resuscitation.

Which instruction should the nurse offer a client as primary preventive measures to prevent mastitis? Avoid massaging the breast area. Avoid frequent breastfeeding. Perform handwashing before breastfeeding. Apply cold compresses to the breast.

Perform handwashing before breastfeeding. Explanation: As a primary preventive measure to prevent mastitis, the nurse should instruct the client to perform good handwashing before breastfeeding. The nurse should instruct the client to frequently breastfeed to prevent engorgement and milk stasis. If the breast is distended before feeding, the nurse should instruct the client to apply cold (not warm) moist heat to the breast. Gently massaging the affected area of the breast also helps.

Amniotic fluid is produced throughout the pregnancy by the fetal membranes. Amniotic fluid has four major functions. What is one of these functions? Medium in which to test organ maturity Physical protection Provide fluid to keep the fetus hydrated Restriction of movement

Physical protection Explanation: Amniotic fluid serves four main functions for the fetus: physical protection, temperature regulation, provision of unrestricted movement, and symmetrical growth.

The nurse is leading a discussion with a group of pregnant women who have diabetes. The nurse should point out which situation can potentially occur during their pregnancy? Small-for-gestational-age (SGA) infant Polyhydramnios Postterm birth Hypotension of pregnancy

Polyhydramnios Explanation: Polyhydramnios is an increase, or excess, in amniotic fluid and is a pregnancy-related complication associated with diabetes. An infant who is small-for-gestational-age is not associated with a mother who had diabetes prior to pregnancy. Other pregnancy-related complications associated with pregestational diabetes mellitus include hypertensive disorders, preterm birth, and shoulder dystocia.

The nurse is assessing a client who believes she is pregnant. The nurse points out a more definitive assessment is necessary due to which sign being considered a probable sign of pregnancy? Fatigue Amenorrhea Positive home pregnancy test Nausea and vomiting

Positive home pregnancy test Explanation: A urine pregnancy test is considered a probable sign of pregnancy as the hCG may be from another source other than pregnancy. Fatigue, amenorrhea, and vomiting are presumptive or possible signs of pregnancy and can also have other causes.

The partner of a pregnant client in her first trimester asks the nurse about the client's behavior recently, stating that she is very moody, seems happy one moment and is crying the next and all she wants to talk about is herself. What response would correctly address these concerns? Her body is changing and she may be angry about it. Pregnant women often experience mood swings and self-centeredness but this is normal. Moodiness and irritability are not usual responses to pregnancy. What you are describing may be normal but we need to talk to her more in depth.

Pregnant women often experience mood swings and self-centeredness but this is normal. Explanation: During the first trimester of pregnancy, the woman often has mood swings, bouts of irritability and is hypersensitive. The partner needs to know that these are all normal behaviors for a pregnant woman.

A woman in labor suddenly reports sharp fundal pain accompanied by slight dark red vaginal bleeding. The nurse should prepare to assist with which situation? Premature separation of the placenta Preterm labor that was undiagnosed Placenta previa obstructing the cervix Possible fetal death or injury

Premature separation of the placenta Explanation: Premature separation of the placenta begins with sharp fundal pain, usually followed by dark red vaginal bleeding. Placenta previa usually produces painless bright red bleeding. Preterm labor contractions are more often described as cramping. Possible fetal death or injury does not present with sharp fundal pain. It is usually painless.

The maternal health nurse is caring for a group of high-risk pregnant clients. Which client condition will the nurse identify as being the highest risk for pregnancy? Secondary hypertension Repaired atrial septal defect Pulmonary hypertension Loud systolic murmur

Pulmonary hypertension Explanation: Pulmonary hypertension is considered the greatest risk to a pregnancy because of the hypoxia that is associated with the condition. The remaining conditions represent potential cardiac complications that may increase the client's risk in pregnancy; however, these do not present the greatest risk in pregnancy.

A G3P2 woman at 39 weeks' gestation presents highly agitated, reporting something "came out" when her membranes just ruptured. Which action should the nurse prioritize after noting the umbilical cord is hanging out of the vagina? Put the client in bed immediately, call for help, and lift the presenting part of the fetus off the cord. With the client in lithotomy position, hold her legs and sharply flex them toward her shoulders. Place the client in Trendelenburg position and gently attempt to reinsert the cord. Contact the health care provider and prepare the client for an emergent vaginal birth.

Put the client in bed immediately, call for help, and lift the presenting part of the fetus off the cord. Explanation: The nurse must put the woman in a bed immediately, while calling for help, and holding the presenting part of the fetus off the cord to ensure its safety. Umbilical cord prolapse occurs when the umbilical cord slips down in front of the presenting part, which can result in the presenting part compressing the cord, cutting off oxygen and nutrients to the baby, and the baby is at risk of death. This is an emergency. When a prolapsed cord is evident the nurse does not put the woman in lithotomy position, and cannot attempt to reinsert the cord. A vaginal birth is contraindicated in this situation.

A 32-year-old woman presents to the labor and birth suite in active labor. She is multigravida, relaxed, and talking with her husband. When examined by the nurse, the fetus is found to be in a cephalic presentation. His occiput is facing toward the front and slightly to the right of the mother's pelvis, and he is exhibiting a flexed attitude. How does the nurse document the position of the fetus? LOA LOP ROA ROP

ROA Explanation: The nurse should document the fetal position in the clinical record using abbreviations. The first letter describes the side of the maternal pelvis toward which the presenting part is facing ("R" for right and "L" for left). The second letter indicates the reference point ("O" for occiput, "Fr" for frontum, etc.). The last part of the designation specifies whether the presenting part is facing the anterior (A) or the posterior (P) portion of the pelvis, or whether it is in a transverse (T) position.

A pregnant woman comes to the emergency department stating she thinks she is in labor. Which assessment finding concerning the pain will the nurse interpret as confirmation that this client is in true labor? Radiates from the back to the front Slows when the woman changes position Occurs in an irregular pattern Lasts about 20 to 25 seconds

Radiates from the back to the front Explanation: Contractions that begin in the back and then radiate to the front are typical of true labor. Contractions that slow when a woman walks or changes position suggest false labor, as do irregular contractions. Contractions lasting 30 seconds or less commonly suggest Braxton Hicks contractions and are associated with false labor.

A client comes to the clinic with concerns about her pregnancy. She is in her first trimester and is now experiencing moderate abdominal pain on the right side. What would be the nurse's first action? Recommend an abdominal ultrasound to the doctor since this may be ectopic pregnancy. Reassure the mother that this is normal as the baby is implanting into the uterus. Obtain a detailed 24-hour intake to determine if the pain is related to what she has eaten. Encourage her to ambulate since gas pains are common in early pregnancy.

Recommend an abdominal ultrasound to the doctor since this may be ectopic pregnancy. Explanation: The nurse should recognize that abdominal pain is not normal during pregnancy and warrants investigation since ectopic pregnancy is a distinct possibility. An abdominal ultrasound would be best practice for this complaint. Dismissing her reports as normal is not a wise choice.

The nurse is caring for a client who received a dose of IV sedation, given by the charge nurse, 30 minutes prior. What action is appropriate? Remind the client to call for assistance before getting out of bed. Restrict the client's fluid to further prevent constipation from the medication. Assure the fetal heart tones are assessed every 2 to 3 hours via monitoring. Remind the client that medication will assist in relieving pain from contractions.

Remind the client to call for assistance before getting out of bed. Explanation: The nurse will remind the client to call for help before getting out of bed to prevent falling from the sedation effects. The nurse would not expect sedatives to cause constipation if given for a limited time during labor. Fetal heart tones are assessed continuously to monitor for side effects of decreased fetal heart rate variability secondary to maternal sedatives. Sedatives do not relieve pain but may provide an opportunity to sleep and decreases anxiety during labor.

A nurse is providing education to a woman at 28 weeks' gestation who has tested positive for gestational diabetes mellitus (GDM). What would be important for the nurse to include in the client teaching? She is at increased risk for type 2 diabetes mellitus after her baby is born. Her baby is at increased risk for neonatal diabetes mellitus. Her baby is at increased risk for type 1 diabetes mellitus. She is at increased risk for type 1 diabetes mellitus after her baby is born.

She is at increased risk for type 2 diabetes mellitus after her baby is born. Explanation: The woman who develops GDM is at increased risk for developing type 2 diabetes mellitus after pregnancy.

The nurse is educating the client at 12 weeks' gestation regarding the best types of exercise throughout pregnancy. Which activities should the nurse encourage? All activities that the client does in a prepregnant state Relaxing activities such as hot baths and jacuzzis High-impact movements enabling less time in the activity Stretching and breathing exercises such as yoga

Stretching and breathing exercises such as yoga Explanation: It is important to exercise during pregnancy. One excellent type of exercise includes yoga, which reduces stress and increases relaxation. Yoga also gently stretches muscles and can increase muscle tone. Contact and high-impact sports are not appropriate for the pregnant mother. Hot areas such as a jacuzzi, hot tub, and sauna are also inappropriate.

The nurse is caring for a client in active labor. Which assessment finding should the nurse prioritize and report to the team? Bradypnea Sudden shortness of breath Bradycardia Unrelieved pain

Sudden shortness of breath Explanation: Sudden shortness of breath can be a sign of amniotic fluid embolism and requires emergent intervention. This can occur suddenly during labor or immediately after. The woman usually develops symptoms of acute respiratory distress, cyanosis, and hypotension. It must be reported to the care team so proper interventions may be taken. Other symptoms can include hypotension, cyanosis, hypoxemia, uterine atony, seizures, tachycardia, coagulation failure, DIC, and pulmonary edema.

The nurse is explaining the discharge instructions to a client who has developed postpartum cystitis. The client indicates she is not drinking a glass of fluid every hour because it hurts too much when she urinates. What is the best response from the nurse? Advise her to take acetaminophen to ease symptoms. Ask primary care provider to prescribe an analgesic. Instruct to use a sitz bath while voiding. Teach that adequate hydration helps clear the infection quicker.

Teach that adequate hydration helps clear the infection quicker. Explanation: Adequate hydration is necessary to dilute the bacterial concentration in the urine and aid in clearing the organisms from the urinary tract. Encourage the woman to drink at least 3000 ml of fluid a day, suggesting she drink one glass per hour. Drinking fluid will make the urine acidic, deterring organism growth. The other choices are also options but address the symptoms and not the root cause. The goal should be to rid the body of the infection, not concentrate on counteracting the results of the infection.

A 1-day-old newborn is being examined by the nurse practitioner, who makes the following notation: face and sclera appear mildly jaundiced. What causes this finding? The breakdown of RBCs release bilirubin, which the liver cannot excrete. The GI tract is immature, so the bilirubin remains in the intestines. The newborn's vitamin K levels are low. Feedings are not adequate to eliminate the build-up of bilirubin.

The breakdown of RBCs release bilirubin, which the liver cannot excrete. Explanation: After birth, the newborn's hematocrit is about 45% to 65%, which is not needed after birth for oxygenation. The cells then die and are broken down, releasing bilirubin. The liver normally breaks down the bilirubin and eliminates it but since the liver is immature, it becomes overwhelmed and the bilirubin builds up in the bloodstream. Vitamin K levels have no effect on bilirubin levels. The immaturity of the GI tract does not cause the bilirubin to increase and feedings do not directly affect bilirubin levels.

The nurse is assessing a 27-year-old woman pregnant for the first time during her initial visit to the obstetric clinic. The client is currently at 8 weeks' gestation. During the assessment, the client reports that she is experiencing sharp stabbing pains in her lower abdomen, especially when she changes position. How would the nurse document this finding? The client is having growing pains. The client is having a spontaneous abortion (miscarriage). The client is having preterm labor. The client is having Braxton Hicks contractions.

The client is having growing pains. Explanation: In the first trimester, sharp pains in the lower abdomen are common. Stretching of the round and broad ligaments that support the growing uterus causes them, which are usually very short (< 5 seconds) and have a stabbing quality. They are not repetitive but are often associated with position changes or, later, fetal movements. The nurse would not document this finding as spontaneous abortion (miscarriage), preterm labor, or Braxton Hicks contractions.

A 20-year-old client gave birth to a baby boy at 43 weeks' gestation. What might the nurse observe in the newborn during routine assessment? The newborn may look wrinkled and old at birth. The infant may have excess of lanugo and vernix caseosa. The testes in the child may be undescended. The newborn may have short nails and hair.

The newborn may look wrinkled and old at birth. Explanation: Postterm babies are those born past 42 weeks' gestation. These babies often appear wrinkled and old at birth. They often have long fingernails and hair, dry parched skin, and no vernix caseosa. Both the quantity of lanugo and the amount of vernix decrease with gestational age. Undescended testes are usually not seen in postterm newborns; however, they are highly prevalent in preterm infants.

The nurse observes a neonate born at 28 weeks' gestation. Which finding would the nurse expect to see? The skin is pale, and no vessels show through it. Creases appear on the interior two-thirds of the sole. The pinna of the ear is soft and flat and stays folded. The neonate has 7 to 10 mm of breast tissue.

The pinna of the ear is soft and flat and stays folded. Explanation: The ear has a soft pinna that is flat and stays folded. Pale skin with no vessels showing through and 7 to 10 mm of breast tissue are characteristic of a neonate at 40 weeks' gestation. Creases on the anterior two-thirds of the sole are characteristic of a neonate at 36 weeks' gestation.

What should the nurse expect for a full-term newborn's weight during the first few days of life? There is an increase in 3% to 5% of birth weight by day 3 in formula-fed babies. A formula-fed newborn should gain 3% to 5% of the initial birth weight in the first 48 hours, but a breastfed newborn may lose up to 3%. There is a loss of 5% to 10% of the birth weight in the first few days in breastfed infants only. There is a loss of 5% to 10% of birth weight in formula-fed and breastfed newborns.

There is a loss of 5% to 10% of birth weight in formula-fed and breastfed newborns. Explanation: The nurse should expect the newborn who is breastfed or formula-fed to lose 5% to 10% of birth weight in the first few days of life.

The nurse instructs the client about skin massage and the gate control theory of pain. Which statement would be appropriate for the nurse to include for client understanding of the nonpharmacologic pain relief methods? The gating mechanism is located at the pain site. Pain perception is decreased if anxiety is present. The gating mechanism opens so all the stimuli pass through to the brain. These methods are a technique to prevent the painful stimuli from entering the brain.

These methods are a technique to prevent the painful stimuli from entering the brain. Explanation: Gate-control diverts the pain stimuli from the pain site by replacing with a comfort stimuli in a new location. Gate control does not need to be applied directly to the site of the pain. Anxiety heightens the painful feelings. Gating blocks the flow of painful stimuli to the sensory centers in the brain.

A client has just given birth to a healthy baby boy, but the placenta has not yet delivered. What stage of labor does this scenario represent? First Second Third Fourth

Third Explanation: Stage three begins with the birth of the baby and ends with delivery of the placenta.

A laboring client is experiencing dysfunctional labor or dystocia due to the malfunction of one or more of the "four Ps" of labor. Which scenario best illustrates a power problem? The fetus is macrosomic. The mother is fighting the contractions. The mother has a small pelvic opening. Uterine contractions are weak and ineffective.

Uterine contractions are weak and ineffective. Explanation: Labor dystocia indicates that the labor is progressing too slowly. Reasons for this are described as due to the "four P's", which are passageway, passenger, power and psyche. A power problem involves either ineffective contractions in either quality or quantity or the mother is too tired to push when needed.

Which procedure is contraindicated in an antepartum client with bright red, painless bleeding? Urinalysis Vaginal examination Leopold maneuver Nonstress test

Vaginal examination Explanation: A vaginal examination is contraindicated in a client with bright red vaginal bleeding until placenta previa is ruled out. The client can have a urinalysis if needed. Leopold maneuver determines fetal position, presentation and attitude. A nonstress test assesses fetal heart rate and movement.

A pregnant client in her second trimester informs the nurse that she needs to travel by air the following week. Which precaution should the nurse instruct the client to take during the flight? Wear a padded bra. Wear low-heeled shoes. Wear support hose. Wear cotton clothes.

Wear support hose. Explanation: The nurse should instruct the client to wear support hose while traveling by air. The nurse should also instruct the client to periodically exercise the legs and ankles, and walk in the aisles if possible. Wearing low-heeled shoes, cotton clothes, or a padded bra will have no effect on the client during the flight.

There has been much research done on pain and the perception of pain. What is the result of research done on levels of satisfaction with the control of labor pain? Women report higher levels of satisfaction when regional anesthetics are used to control pain. Women report higher levels of satisfaction when they felt they had a high degree of control over the pain experience. Women report higher levels of satisfaction when the primary care provider makes the decision on what type of pain control to use. Women report higher levels of satisfaction when different types of relaxation techniques are used to control pain.

Women report higher levels of satisfaction when they felt they had a high degree of control over the pain experience. Explanation: Women identify being involved in their pain management and adequate control of their pain as important factors in their overall labor experience. Women often report that it is not the amount of pain they have during labor that contributes to a satisfactory birth experience but rather how their pain is managed.

A client states that "she thinks" her water has broken. Which best provides confirmation of the rupture of membranes? leakage from the perineum when the client coughs greenish fluid noted on the client's underwear a positive bacterial culture a positive nitrazine test

a positive nitrazine test Explanation: A confirmation that the client has a rupture of membranes includes a positive nitrazine test. A positive test is when the nitrazine paper turns a dark blue indicating that the fluid is alkaline. Urine also leaks when a client coughs. Greenish fluid on the underwear is not confirmation of the rupture of membranes. A positive bacterial culture is not indicative of the rupture of membranes.

A woman in her second trimester of pregnancy is beginning to experience more headaches. In addition to suggesting holding an ice pack to the forehead, the health care provider recommends which medication to provide some relief from the pain? ibuprofen aspirin products acetaminophen naproxen

acetaminophen Explanation: Resting with an ice pack on the forehead and taking a usual adult dose of acetaminophen usually furnishes adequate relief. Compounds with ibuprofen (class C drugs) are not usually recommended because they cause premature closure of the ductus arteriosus in the fetus. Additionally, they have been found to contribute to fetal renal damage, low amniotic fluid, and fetal intracranial hemorrhage. Aspirin and naproxen are also not recommended to take during pregnancy.

Immediately after giving birth to a full-term infant, a client develops dyspnea and cyanosis. Her blood pressure decreases to 60/40 mm Hg, and she becomes unresponsive. What does the nurse suspect is happening with this client? placental separation aspiration amniotic fluid embolism congestive heart failure

amniotic fluid embolism Explanation: With amniotic fluid embolism, symptoms may occur suddenly during or immediately after labor. The woman usually develops symptoms of acute respiratory distress, cyanosis, and hypotension.

A woman who is 8 months pregnant comes to the clinic with urinary frequency and pain on urination. The client is diagnosed with a urinary tract infection (UTI). Which medication would the nurse anticipate the physician will prescribe? tetracycline amoxicillin bactrim septra

amoxicillin Explanation: Amoxicillin is a penicillin antibiotic and can be used in the pregnant woman to treat a UTI. Tetracycline should never be given to a pregnant woman, because it may cause retardation of bone growth and staining of the fetal teeth. The sulfonamides (bactrim and septra) can be used in early pregnancy but not near term, because they can interfere with protein binding of bilirubin, which then can lead to hyperbilirubinemia in the newborn.

The clinic nurse teaches a client with pregestational type 1 diabetes that maintaining a constant insulin level is very important during pregnancy. The nurse tells the client that the best way to maintain a constant insulin level is to use: regular insulin twice a day. an insulin pen. an insulin drip. an insulin pump.

an insulin pump. Explanation: Because a pregnant client will have some periods of relative hyperglycemia and hypoglycemia no matter how carefully the client maintains diet and balances exercise levels, an effective method to keep serum glucose levels constant is to administer insulin with a continuous pump during pregnancy.

A pregnant woman with a history of mitral valve stenosis is to be prescribed medication as treatment. Which medication class would the nurse expect the client to be prescribed? anticoagulant vasodilator inotropic angiotensin receptor blockers

anticoagulant Explanation: In mitral valve stenosis, it is difficult for blood to leave the left atrium. A secondary problem of thrombus formation may develop as a result of noncirculating blood. A woman may need to be prescribed an anticoagulant to prevent this complication. Vasodilators are used for peripartum cardiomyopathy. Inotropics are used for heart failure. Angiotensin receptor blockers are used for congestive heart failure.

A woman's obstetrician prescribes vitamin K supplements for a client who is on antiepileptic medications beginning at 36 weeks' gestation. The mother asks the nurse why she is taking this medication. The nurse's best response would be: vitamin K helps in keeping the placenta healthy. antiepileptic therapy can lead to vitamin K-deficient hemorrhage of the newborn. administration of vitamin K aids in lung maturity of the fetus. The antiepileptic medications can cause the mother's platelets to drop.

antiepileptic therapy can lead to vitamin K-deficient hemorrhage of the newborn. Explanation: Antiepileptic therapy may cause vitamin K-deficient hemorrhage of the newborn. The vitamin K injection the newborn receives following birth can't fully correct this, so some physicians recommend a vitamin K supplement for their pregnant clients beginning at 36 weeks' gestation. If the mother should go into preterm labor, the newborn will have received the vitamin K prior to delivery. However, many physicians now question the usefulness of the prophylaxis.

The nurse institutes measures to maintain thermoregulation based on the understanding that newborns have limited ability to regulate body temperature because they: have a smaller body surface compared to body mass. lose more body heat when they sweat than adults. have an abundant amount of subcutaneous fat all over. are unable to shiver effectively to increase heat production.

are unable to shiver effectively to increase heat production. Explanation: Newborns have difficulty maintaining their body heat through shivering and other mechanisms. They have a large body surface area relative to body weight and have limited sweating ability. Additionally, newborns lack subcutaneous fat to provide insulation.

The nurse is examining a woman who came to the clinic because she thinks she is pregnant. Which data collected by the nurse are presumptive signs of her pregnancy? Select all that apply. breast changes ultrasound pictures fetal heartbeat amenorrhea hydatidiform mole morning sickness

breast changes amenorrhea morning sickness Presumptive signs are possible signs of pregnancy that appear in the first trimester, often only noted subjectively by the mother (e.g., breast changes, amenorrhea, morning sickness). Probable signs are signs that appear in the first and early second trimesters, seen via objective criteria, but can also be indicators of other conditions (e.g., hydatidiform mole). Positive signs affirm that proof exists that there is a developing fetus in any trimester and are objective criteria seen by a trained observer or diagnostic study, (e.g., ultrasound.)

The nurse is teaching a primigravida who does not speak the dominant language. The nurse will teach about the most common type of fetal presentation. Which presentation will the nurse prepare? breech presentation using a picture cephalic presentation using preprinted materials in the client's language occiput presentation using a PowerPoint presentation footling presentation drawing a hand-prepared diagram

cephalic presentation using preprinted materials in the client's language Explanation: The most common presentation type is the cephalic presentation, and it is most appropriate to highlight the information using preprinted materials in the client's language. Both portions of this answer are best. With pictures, the nurse can communicate on a common level and then the client has the opportunity to review as needed. The breech and occiput presentations are not the most common types of fetal presentation. The footling is not a type of fetal presentation.

A nurse is providing care to a pregnant woman in labor. The woman is in the first stage of labor. When describing this stage to the client, which event would the nurse identify as the major change occurring during this stage? regular contractions cervical dilation (dilatation) fetal movement through the birth canal placental separation

cervical dilation (dilatation) Explanation: The primary change occurring during the first stage of labor is progressive cervical dilation (dilatation). Contractions occur during the first and second stages of labor. Fetal movement through the birth canal is the major change during the second stage of labor. Placental separation occurs during the third stage of labor.

A pregnant woman with diabetes at 10 weeks' gestation has a glycosylated hemoglobin (HbA1c) level of 13%. At this time the nurse should be most concerned about which possible fetal outcome? congenital anomalies incompetent cervix placenta previa placental abruption (abruptio placentae)

congenital anomalies Explanation: A HbA1c level of 13% indicates poor glucose control. This, in conjunction with the woman being in the first trimester, increases the risk for congenital anomalies in the fetus. Elevated glucose levels are not associated with incompetent cervix, placenta previa, or placental abruption (abruptio placentae).

A Black couple are spending time with their newborn after the nurse brings the newborn back from the transition nursery. The parents note that their newborn's buttocks appear bruised and ask what happened. The nurse should explain this is related to which factor? lanugo vascular nevi bruising congenital dermal melanocytosis (slate gray nevi)

congenital dermal melanocytosis (slate gray nevi) Explanation: The mark described is congenital dermal melanocytosis (slate gray nevi, previously known as Mongolian spots), which occurs in ethnicities with darker colored skin. This is a normal finding that should disappear within one to tow years. It does require documentation. Lanugo is the fine hair on the newborn's body when it is born. Vascular nevi are birthmarks. Although the slate gray nevi may look like bruising, they are not.

The nursing instructor is preparing a class presentation covering the various hormones and their functions during pregnancy. The instructor determines the class is successful when the class correctly matches which function with hCG? provides rich blood supply to decidua maintains nutrient-rich decidua continues progesterone production by corpus luteum sustains life of placenta

continues progesterone production by corpus luteum Explanation: The corpus luteum is responsible for producing progesterone until this function is assumed by the placenta. hCG is a fail-safe mechanism to prolong the life of the corpus luteum and ensure progesterone production. Estrogen is responsible for providing a rich blood supply to the decidua. Progesterone helps maintain a nutrient-rich decidua.

Which intervention would be most important when caring for the client with breech presentation confirmed by ultrasound? applying suprapubic pressure against the fetal back continuing to monitor maternal and fetal status noting the space at the maternal umbilicus auscultating the fetal heart rate at the level of the umbilicus

continuing to monitor maternal and fetal status Explanation: Once a breech presentation is confirmed by ultrasound, the nurse should continue to monitor the maternal and fetal status when the team makes decisions about the method of birth. The nurse usually plays an important role in communicating information during this time. Applying suprapubic pressure against the fetal back is the nursing intervention for shoulder dystocia and may not be required for breech presentation. Noting the space or dip at the maternal umbilicus and auscultating the fetal heart rate at the umbilicus level are assessments related to occipitoposterior positioning of the fetus.

During pregnancy a woman has many psychological adaptations that must be made. The nurse must remember that the baby's father is also experiencing the pregnancy and has adaptations that must be made. Some fathers actually have symptoms of the pregnancy along with the mothers. What is this called? pseudo pregnancy pregnancy syndrome couvade syndrome cretinism

couvade syndrome Explanation: Some fathers actually experience some of the physical symptoms of pregnancy, such as nausea and vomiting, along with their partner. This phenomenon is called couvade syndrome.

A client is diagnosed with gestational hypertension and is receiving magnesium sulfate. The nurse determines that the medication is at a therapeutic level based on which finding? urinary output of 20 mL per hour respiratory rate of 10 breaths/minute deep tendons reflexes 2+ difficulty in arousing

deep tendons reflexes 2+ Explanation: With magnesium sulfate, deep tendon reflexes of 2+ would be considered normal and therefore a therapeutic level of the drug. Urinary output of less than 30 mL, a respiratory rate of less than 12 breaths/minute, and a diminished level of consciousness would indicate magnesium toxicity.

It is most likely that the practitioner would consider performing an amnioinfusion if the EFM tracing shows which of the following? consistent early accelerations, variability present, and occasional decelerations flat line without variability and no decelerations occasional mild variable decelerations and moderate variability present deep variable decelerations more than 60 bpm below the baseline with every contraction

deep variable decelerations more than 60 bpm below the baseline with every contraction Explanation: Repetitive variable decelerations with loss of variability, or ones that last longer than one minute, or dip deeper than 60 bpm below the baseline are nonreassuring.

The student nurse is learning about normal labor. The teacher reviews the cardinal movements of labor and determines the instruction has been effective when the student correctly states the order of the cardinal movements as follows: internal rotation, descent, extension, flexion, external rotation, expulsion descent, flexion, external rotation, extension, internal rotation, expulsion descent, flexion, internal rotation, extension, external rotation, expulsion internal rotation, flexion, descent, extension, external rotation, expulsion

descent, flexion, internal rotation, extension, external rotation, expulsion Explanation: The six cardinal movements of the fetus, in order, are descent, flexion, internal rotation, extension, external rotation, and expulsion.

A nurse is performing a physical assessment of a woman in labor. As part of her assessment, she examines the outer and inner surfaces of her lips. What is the best rationale for this assessment? detection of herpes virus infection detection of a respiratory infection detection of anemia detection of rales

detection of herpes virus infection Explanation: Examine the outer and inner surfaces of her lips carefully to detect herpes lesions (pinpoint vesicles on an erythematous base). Report to her primary care provider if herpetic lesions are present anywhere because although oral lesions are invariably a type I herpes virus (common cold sores), type II (genital) herpes virus needs to be identified as this can be lethal to newborns; a woman primary health care provider may suggest the woman with oral herpes lesions take isolation precautions such as not kissing her newborn until the lesions crust. Be certain to palpate for enlargement of neck lymph nodes to detect the possibility of a respiratory infection. Inspect the mucous membrane of her mouth and the conjunctiva of her eyes for color to see if paleness suggests anemia. Auscultate the woman's lungs to be certain they are clear of rales.

A 29-year-old client has gestational diabetes. The nurse is teaching her about managing her glucose levels. Which therapy would be most appropriate for this client? diet long-acting insulin oral hypoglycemic drugs glucagon

diet Explanation: Clients with gestational diabetes are usually managed by diet alone to control their glucose intolerance. Long-acting insulin usually is not needed for blood glucose control in the client with gestational diabetes. Oral hypoglycemic drugs are usually not given during pregnancy and would not be the first option. Glucagon raises blood glucose and is used to treat hypoglycemic reactions.

During a prenatal visit, the health care provider determines that the fetal heart beat is too fast. What drug would the nurse expect to be prescribed to the mother to treat the concern? adenosine digoxin lisinopril nitroglycerine

digoxin Explanation: Digoxin is sometimes administered to a woman during pregnancy to slow the fetal heart rate if fetal tachycardia is present.

A multipara client develops thrombophlebitis after birth. Which assessment findings would lead the nurse to intervene immediately? dyspnea, diaphoresis, hypotension, and chest pain dyspnea, bradycardia, hypertension, and confusion weakness, anorexia, change in level of consciousness, and coma pallor, tachycardia, seizures, and jaundice

dyspnea, diaphoresis, hypotension, and chest pain Explanation: Sudden unexplained shortness of breath and reports of chest pain along with diaphoresis and hypotension suggest pulmonary embolism, which requires immediate action. Other signs and symptoms include tachycardia, apprehension, hemoptysis, syncope, and sudden change in the woman's mental status secondary to hypoxemia. Anorexia, seizures, and jaundice are unrelated to a pulmonary embolism.

Current research indicates that supplementation with what before pregnancy may reduce the risk of placental abruption? vitamin C iron folic acid calcium

folic acid Explanation: New research indicates that folic acid supplementation before or during pregnancy reduces the risk of placental abruption. Neither supplementation with vitamin C, iron, nor calcium is associated with a decreased risk for placental abruption.

A nurse is caring for a 33-year-old primigravida client who is obese and near the end of their second trimester. The client has a history of prepregnancy obesity, hypertension, and smoking. Complete the following sentence(s) by choosing from the lists of options. The client is at highest risk for developing Select...gestational diabetesectopic pregnancygestational trophoblastic diseasespontaneous abortionSelect.... The nurse provides discharge teaching to reduce the risks of developing this condition. Teaching should include Select...change in lifestylelimit smokinglimit exercise during pregnancyrefrain from having intercourse during pregnancySelect....

gestational diabetes and lifestyle change

Which nursing action is required before a client in labor receives an epidural? giving a fluid bolus of 500 ml checking for maternal pupil dilation testing maternal reflexes observing maternal gait

giving a fluid bolus of 500 ml Explanation: One of the major adverse effects of epidural administration is hypotension. Therefore, a 500-ml fluid bolus is usually administered to help prevent hypotension in the client who wishes to receive an epidural for pain relief. Checking maternal reflexes, pupil response, and gait aren't necessary.

A pregnant client tells the nurse, "My health care provider told me that I have the best type of pelvic shape to deliver my baby vaginally." The nurse interprets this statement as indicating that the client has which pelvic shape? platypelloid gynecoid android anthropoid

gynecoid Explanation: A gynecoid, or "female," pelvis has an inlet that is well rounded forward and backward and has a wide pubic arch. This pelvic type is ideal for childbirth. In an android, or "male," pelvis, the pubic arch forms an acute angle, making the lower dimensions of the pelvis extremely narrow. A fetus may have difficulty exiting from this type of pelvis. In an anthropoid, or "ape-like," pelvis, the transverse diameter is narrow; the anteroposterior diameter of the inlet is larger than usual. Even though the inlet is large, the shape of the pelvis does not accommodate a fetal head as well as a gynecoid pelvis. A platypelloid, or "flattened," pelvis has a smoothly curved oval inlet, but the anteroposterior diameter is shallow. A fetal head might not be able to rotate to match the curves of the pelvic cavity.

A client has given birth to a small-for-gestational-age (SGA) newborn. Which finding would the nurse expect to assess? head larger than body round flushed face brown lanugo body hair protuberant abdomen

head larger than body Explanation: A small-for-gestational-age (SGA) newborn will typically have a head that is larger than the rest of his or her body. SGA newborns weigh below the 10th percentile on the intrauterine growth chart for gestational age. They have an angular and pinched face and not a rounded and flushed face. Round flushed face and protuberant abdomen are the characteristic features of large-for-gestational-age (LGA) newborns. Preterm newborns, and not SGA newborns, are covered with brown lanugo hair all over the body.

A client in active labor is given spinal anesthesia. Which information would the nurse include when discussing with the client and family about the disadvantages of spinal anesthesia? passage of the drug to the fetus headache following anesthesia excessive contractions of the uterus increased frequency of micturition

headache following anesthesia Explanation: The nurse should inform the client and her family about the possibility of headache after spinal anesthesia. The drug is retained in the mother's body and not passed to the fetus. There may be uterine atony, and not excessive uterine contractions, following spinal anesthesia. Spinal anesthesia may lead to bladder atony, and not an increased frequency of micturition.

The Apgar score is based on which 5 parameters? heart rate, muscle tone, reflex irritability, respiratory effort, and color heart rate, breaths per minute, irritability, reflexes, and color heart rate, respiratory effort, temperature, tone, and color heart rate, breaths per minute, irritability, tone, and color

heart rate, muscle tone, reflex irritability, respiratory effort, and color Explanation: A newborn can receive an Apgar score ranging from 0 to 10. The score is based on 5 factors, each of which is assigned a 0, 1, or 2. Heart rate (should be above 100), muscle tone (should be able to maintain a flexion position), reflex irritability (newborn should cry or sneeze when stimulated), and respiratory effort are evaluated by the presence of a strong cry and by color. Color is evaluated by noting the color of the body and hands and feet.

A client asks the nurse what surfactant is. Which explanation would the nurse give as the main role of surfactant in the neonate? assists with ciliary body maturation in the upper airways helps maintain a rhythmic breathing pattern promotes clearing of mucus from the respiratory tract helps the lungs remain expanded after the initiation of breathing

helps the lungs remain expanded after the initiation of breathing Explanation: Surfactant works by reducing surface tension in the lung, which allows the lung to remain slightly expanded, decreasing the amount of work required for inspiration. Surfactant has not been shown to influence ciliary body maturation, clearing of the respiratory tract, or regulation of the neonate's breathing pattern.

A pregnant client at 8 weeks' gestation comes to the facility for vaginal bleeding. Assessment reveals that the client has experienced an incomplete spontaneous abortion (miscarriage) for which suction curettage is planned. While preparing the client for the procedure, the nurse would closely monitor for which possible complication? isoimmunization hemorrhage gestational hypertension HELLP syndrome

hemorrhage Explanation: With an incomplete miscarriage, there is a danger of maternal hemorrhage as long as part of the conceptus is retained in the uterus because the uterus cannot contract effectively under this condition. Gestational hypertension or HELLP syndrome (Hemolysis, Elevated Liver enzymes, and Low Platelets) are not associated with an incomplete miscarriage. It would be important to determine the client's Rh type because the blood type of the conceptus is unknown. Therefore, all clients with Rh-negative blood should receive Rh (D antigen) immune globulin (RhIG) to prevent the buildup of antibodies in the event the conceptus was Rh-positive to prevent isoimmunization in future pregnancies.

A nurse is caring for a 25-year-old G1P0 at 37 weeks' gestation. The client's history indicates that the client has had alcohol abuse disorder throughout their pregnancy. What signs and symptoms does the nurse expect the newborn to exhibit? Drag words from the choices below to fill in each blank in the following sentence. The nurse recognizes the newborn is exhibiting signs and symptoms of fetal alcohol syndrome, which include , , , and . Signs/Symptomshigh-pitched, shrill cryincreased appetitejitterinesseasily consoledlarge, narrow-spaced eyesflat midfacethin upper lip

high-pitched, shrill cry, jitteriness, flat midface, thin upperlip Fetal alcohol syndrome (FAS) is caused by intake of alcohol by the pregnant parent during pregnancy; alcohol consumption may be periodic or chronic. Newborns born with FAS have characteristic facial features, are more susceptible to congenital defects, and often have developmental delays. Newborns with FAS have a high-pitched and shrill cry and are generally jittery. Newborns with FAS have a flat midface and a thin upper lip. Newborns with FAS have small and wide-spaced eyes not large narrow-spaced eyes. Newborns with FAS are not easily consoled and have a poor, not an increased appetite.

A woman at 8 weeks' gestation is admitted for ectopic pregnancy. She is asking why this has occurred. The nurse knows that which factor is a known risk factor for ectopic pregnancy? high number of pregnancies multiple gestation pregnancy use of oral contraceptives history of endometriosis

history of endometriosis Explanation: The nurse needs to complete a full history of the client to determine if she had any other risk factors for an ectopic pregnancy. Adhesions, scarring, and narrowing of the tubal lumen may block the zygote's progress to the uterus. Any condition or surgical procedure that can injure a fallopian tube increases the risk. Examples include salpingitis, infection of the fallopian tube, endometriosis, history of prior ectopic pregnancy, any type of tubal surgery, congenital malformation of the tube, and multiple abortions (elective terminations of pregnancy). Conditions that inhibit peristalsis of the tube can result in tubal pregnancy. A high number of pregnancies, multiple gestation pregnancy, and the use of oral contraceptives are not known risk factors for ectopic pregnancy.

The client plans to undergo a vaginal birth after cesarean (VBAC) and has recently begun labor. The labor nurse is following the fetal monitor closely. The results of which monitoring will best help the nurse assess contraction strength in the client? internal electronic monitoring maternal heart rate monitoring fetal oxygen saturation level fetal blood sampling

internal electronic monitoring Explanation: A client who is undergoing a vaginal birth after cesarean (VBAC) is at a high risk for uterine rupture. For that reason, the nurse will follow uterine contraction strength. Internal electronic monitoring is the most precise method for assessing fetal heart rate (FHR) and uterine contractions. As each uterine contraction puts pressure on the uterine contents, the pressure exerted on the catheter is recorded. When contractions are monitored by an internal pressure gauge in this way, the frequency, duration, baseline strength, and peak strength of contractions can all be evaluated. If FHR variability appears to be depressed during labor, the welfare of a fetus can be assessed by scalp stimulation. Maternal heart rate changes during contractions depending on pain level. Fetal oxygen saturation level and fetal blood sampling are performed to assess the amount of oxygen in the fetal blood.

The nurse is caring for a client who is receiving epidural analgesia with morphine following cesarean birth. For which side effects should the nurse monitor? Select all that apply. itching nausea vomiting hyperventilation gastrointestinal bleeding

itching nausea vomiting Intense itching, nausea, and vomiting can occur following epidural morphine administration. These side effects can be managed with antihistamines for itching and antiemetics for nausea. Hyperventilation and gastrointestinal bleeding are not expected side effects of epidural morphine administration.

Early in labor, a pregnant client asks why contractions hurt so much. Which answer should the nurse provide? lack of oxygen to the muscle fibers of the uterus due to compression of blood vessels release of endorphins in response to the uterine contractions distraction of the brain cortex by other stimuli occuring in the body blocking of nerve transmission via mechanical irritation of nerve fibers

lack of oxygen to the muscle fibers of the uterus due to compression of blood vessels Explanation: During contractions, blood vessels constrict, reducing the blood supply to uterine and cervical cells, resulting in anoxia to muscle fibers. This anoxia can cause pain in the same way blockage of the cardiac arteries causes the pain of a heart attack. Endorphins are naturally occurring opiate-like substances that reduce pain, not cause it. Distraction and mechanical irritation of nerve fibers are also methods of reducing pain, not causes of pain.

The nurse is monitoring a client who is in labor and notes the client is happy, cheerful, and "ready to see the baby." The nurse interprets this to mean the client is in which stage or phase of labor? active stage two latent stage three

latent Explanation: The client in labor undergoes numerous psychologic adaptations during labor. During the latent phase, the client is often talkative and happy, and yet anxious. During active phase, the client may show fear and anger. Both the latent and active phases occur during stage 1. During stages 2 and, the client may remain positive, but the work of labor is very intense.

Assessment of a client in labor reveals cervical dilation of 3 cm, cervical effacement of 30%, and contractions occurring every 7 to 8 minutes, lasting about 40 seconds. The nurse determines that this client is in: latent phase of the first stage. active phase of the first stage. pelvic phase of the second stage. early phase of the third stage.

latent phase of the first stage. Explanation: The latent phase of the first stage of labor involves cervical dilation of 0 to 3 cm, cervical effacement of 0% to 40%, and contractions every 5 to 10 minutes lasting 30 to 45 seconds. The active phase is characterized by cervical dilation of 4 to 7 cm, effacement of 40% to 80%, and contractions occurring every 2 to 5 minutes lasting 45 to 60 seconds. The perineal phase of the second stage occurs with complete cervical dilation and effacement, contractions occurring every 2 to 3 minutes and lasting 60 to 90 seconds, and a tremendous urge to push by the pregnant client. The third stage, placental expulsion, starts after the newborn is born and ends with the separation and birth of the placenta.

The student nurse is preparing to assess the fetal heart rate (FHR) and has determined that the fetal back is located toward the client's left side, the small parts toward the right side, and there is a vertex (occiput) presentation. The nurse should initially begin auscultation of the fetal heart rate in the mother's: right upper quadrant. right lower quadrant. left upper quadrant. left lower quadrant.

left lower quadrant. Explanation: The best position to auscultate fetal heart tones in on the fetus back. In this position, the best place for the FHR monitor is on the left lower quadrant.

The nurse assesses a client in labor and finds that the fetal long axis is longitudinal to the maternal long axis. How should the nurse document this finding? presentation attitude lie position

lie Explanation: The nurse is assessing fetal lie, the relationship of the fetal long axis to the maternal long axis. When the fetal long axis is longitudinal to the maternal long axis, the lie is said to be longitudinal. Presentation is the portion of the fetus that overlies the maternal pelvic inlet. Attitude is the relationship of the different fetal parts to one another. Position is the relationship of the fetal denominator to the different sides of the maternal pelvis.

Increased pigmentation on the face of some pregnant women is called: nigra melanotropin striae gravidarum (stretch marks) melasma (chloasma)

melasma (chloasma) Explanation: Melasma (chloasma), or "mask of pregnancy," is a blotchy brown discoloration on the face. In some women, a darkened line up the abdomen appears, which is called linea nigra. Striae gravidarum are "stretch marks. Melanotropin is the hormone responsible for melasma (chloasma).

Which medication will the nurse anticipate the health care provider will prescribe as treatment for an unruptured ectopic pregnancy? oxytocin promethazine ondansetron methotrexate

methotrexate Correct response: methotrexate Explanation: Methotrexate, a folic acid antagonist that inhibits cell division in the developing embryo, is most commonly used to treat ectopic pregnancy. Oxytocin is used to stimulate uterine contractions and would be inappropriate for use with an ectopic pregnancy. Promethazine and ondansetron are antiemetics that may be used to treat hyperemesis gravidarum.

A full-term neonate delivered an hour after the mother received IV meperidine is showing signs of respiratory depression. The nurse should be prepared to administer which medication? indomethacin ampicillin naloxone epinephrine

naloxone Explanation: Naloxone is the drug used for reversal of opioids' adverse effects. If an opioid is given too close to birth, the fetal liver takes 2 to 3 hours to activate a drug, the effect will not be registered (respiratory depression, sleepiness) in the fetus for 2 to 3 hours after birth. Indomethacin is an analgesic and NSAID; ampicillin is an antibiotic; and epinephrine is a vasopressor.

A client expresses concern that her 2-hour-old newborn is sleepy and difficult to awaken. The nurse explains that this behavior indicates: normal progression of behavior. probable hypoglycemia. physiological abnormality. inadequate oxygenation.

normal progression of behavior. Explanation: From 30 to 120 minutes of age, the newborn enters the second stage of transition, that of sleep or a decrease in activity. More information would be needed to determine if hypoglycemia, a physiologic abnormality, or inadequate oxygenation was present.

Assessment reveals that the fetus of a client in labor is in the vertex presentation. The nurse determines that which part is presenting? shoulders occiput brow buttocks

occiput Explanation: With a vertex presentation, a type of cephalic presentation, the fetal presenting part is the occiput. The shoulders are the presenting part when the fetus is in a shoulder presentation. The brow or sinciput is the presenting part when a fetus is in a brow presentation. The buttocks are the presenting part when a fetus is in a breech presentation.

The nurse is caring for a client who required a forceps-assisted birth. For which potential factor should the nurse be alert? increased risk for uterine rupture potential lacerations and bleeding increased risk for cord entanglement damage to the pregnant client's tissues

potential lacerations and bleeding Explanation: Forcible rotation of the forceps can cause potential lacerations and bleeding. Cervical ripening increases the risk for uterine rupture in a client attempting vaginal birth after undergoing at least one previous cesarean birth. There is an increased risk for cord entanglement in multiple pregnancies. Damage to the pregnant client's tissues happens if the cup slips off the fetal head and the suction is not released during a vacuum-assisted birth.

A client pregnant with twins comes to the clinic for an evaluation. While assessing the client, the nurse is especially alert for signs and symptoms for which potential concern? oligohydramnios preeclampsia postterm labor chorioamnionitis

preeclampsia Explanation: Clients with multiple gestation are at high risk for preeclampsia, preterm labor, polyhydramnios, hyperemesis gravidarum, anemia, and antepartal hemorrhage. There is no association between multiple gestation and the development of chorioamnionitis.

Thick vernix and absence of testicular rugae are a characteristics of

prematurity.

A woman at 35 weeks' gestation with severe polyhydramnios is admitted to the hospital. The nurse recognizes that which concern is greatest regarding this client? preterm rupture of membranes followed by preterm birth development of eclampsia hemorrhaging development of gestational trophoblastic disease

preterm rupture of membranes followed by preterm birth Explanation: Even with precautions, in most instances of polyhydramnios, there will be preterm rupture of the membranes because of excessive pressure, followed by preterm birth. The other answers are less concerning than preterm birth in this pregnancy.

A 24-year-old client presents in labor. The nurse notes there is an order to administer Rho(D) immune globulin after the birth of her infant. When asked by the client the reason for this injection, which reason should the nurse point out? promote maternal D antibody formation. prevent maternal D antibody formation. stimulate maternal D immune antigens. prevent fetal Rh blood formation.

prevent maternal D antibody formation. Explanation: Because Rho(D) immune globulin contains passive antibodies, the solution will prevent the woman from forming long-lasting antibodies which may harm a future fetus. The administration of Rho(D) immune globulin does not promote the formation of maternal D antibodies; it does not stimulate maternal D immune antigens or prevent fetal Rh blood formation.

A client in her fifth month of pregnancy is having a routine clinic visit. The nurse should assess the client for which common second trimester condition? mastitis metabolic alkalosis pseudoanemia respiratory acidosis

pseudoanemia Explanation: Because the blood volume expands during pregnancy slightly ahead of the red cell count, most women have a pseudoanemia in early pregnancy. This condition is normal and should not be confused with true types of anemia that occur as complications of pregnancy. Mastitis is an infection in the breast characterized by a swollen tender breast and flu-like symptoms. This condition is most commonly seen in breastfeeding clients. Alterations in acid-base balance during pregnancy result in a state of respiratory alkalosis, compensated by mild metabolic acidosis.

A nurse is reviewing a client's history and physical examination findings. Which information would the nurse identify as contributing to the client's risk for an ectopic pregnancy? use of oral contraceptives for 5 years ovarian cyst 2 years ago recurrent pelvic infections heavy, irregular menses

recurrent pelvic infections Explanation: In the general population, most cases of ectopic pregnancy are the result of tubal scarring secondary to pelvic inflammatory disease. Oral contraceptives, ovarian cysts, and heavy, irregular menses are not considered risk factors for ectopic pregnancy.

A client is diagnosed with peripartum cardiomyopathy (PPCM). Which therapy should the nurse expect to administer to the client? monoamine oxidase inhibitors (MAOIs) methadone therapy restricted sodium intake ginger therapy

restricted sodium intake Explanation: The client with peripartum cardiomyopathy should be prescribed a restricted sodium intake to control their blood pressure. Monoamine oxidase inhibitors are given to treat depression in pregnancy, not peripartum cardiomyopathy. Methadone is given for the treatment of a substance use disorder during pregnancy. Complementary therapies like ginger therapy help in the alleviation of hyperemesis gravidarum, not peripartum cardiomyopathy.

Untreated hyperemesis can lead to preterm birth. What is the cause of the preterm birth? severe dehydration resulting in hypoperfusion of the placenta ketonuria resulting in neurologic changes in the fetus poor nutrient intake resulting in poor fetal growth class B drugs used to control the vomiting resulting in uterine contractions

severe dehydration resulting in hypoperfusion of the placenta Explanation: With severe dehydration there is hypoperfusion to the placenta, and preterm labor may be initiated. Ketonuria impacts the fetus' neurologic development but does not initiate preterm labor. Medications used to control nausea and vomiting do not induce labor.

A woman at 12 weeks' gestation comes to the clinic with vaginal bleeding. When assessing the woman further, the nurse would suspect a threatened abortion based on which finding? slight vaginal bleeding cervical dilation high beta human chorionic gonadotropin (hCG) level passage of fetal tissue

slight vaginal bleeding Explanation: Slight vaginal bleeding early in pregnancy, no cervical dilation, and a closed cervical os are associated with a threatened abortion. With an inevitable abortion, passage of the products of conception may occur. No fetal tissue is passed with a threatened abortion. Beta hCG is a hormone produced by trophoblast cells of the placenta, and its levels double every 29 to 53 hours during the first 30 days post implantation with a healthy, progressing pregnancy.

A client with epilepsy who is taking phenytoin sodium has just learned that they are pregnant. When planning this client's care, which intervention will the nurse include? teaching about vitamin K during the last 4 weeks of pregnancy discontinuing use of phenytoin sodium adding a second antiseizure medication preparing the client for a cesarean birth

teaching about vitamin K during the last 4 weeks of pregnancy Explanation: The goal of care for a client with a seizure disorder, such as epilepsy, is to establish the best seizure control with the fewest possible number of antiseizure drugs prior to pregnancy. The nurse will need to be certain that the client understands the rule "Do not take medication during pregnancy" does not apply to antiseizure medications, so the client should continue to conscientiously take this medication despite the nausea or vomiting of early pregnancy. To counteract the vitamin K deficiency associated with phenytoin sodium use and to prevent hemorrhage in the newborn, the client may be prescribed vitamin K during labor or the last 4 weeks of gestation. A cesarean birth is not necessary for a client with epilepsy.

Review of a primiparous woman's labor and birth record reveals a prolonged second stage of labor and extended time in the stirrups. Based on an interpretation of these findings, the nurse would be especially alert for which condition? retained placental fragments hypertension thrombophlebitis uterine subinvolution

thrombophlebitis Explanation: The woman is at risk for thrombophlebitis due to the prolonged second stage of labor, necessitating an increased amount of time in bed, and venous pooling that occurs when the woman's legs are in stirrups for a long period of time. These findings are unrelated to retained placental fragments, which would lead to uterine subinvolution, or hypertension.

A young client gives birth to twin boys who shared the same placenta. For what serious complication should the nuse prepare? twin-to-twin transfusion syndrome (TTTS) hemolysis, elevated liver enzymes and low platelets (HELLP) syndrome toxoplasmosis. other, rubella, cytomegalovirus, and genital herpes (TORCH) infection ABO incompatibility

twin-to-twin transfusion syndrome (TTTS) Explanation: When twins share a placenta, a serious condition called twin-to-twin transfusion syndrome (TTTS) can occur. HELLP syndrome (which stands for hemolysis, elevated liver enzymes, low platelets count) is a hypertensive disorder of pregnancy that occurs most often as a complication of preeclampsia. TORCH infection is a group of infections commonly implicated in congenital anomalies. It stands for toxoplasmosis, other, rubella, cytomegalovirus, and genital herpes. ABO incompatibility is a condition where the newborn's blood type is different from the gestational parent's blood type. If the two blood types mix, the newborn's body may attack the red blood cells and break them down.

When assessing the newborn's umbilical cord, what should the nurse expect to find? two smaller arteries and one larger vein two smaller veins and one larger artery one smaller vein and two larger arteries one smaller artery and two larger veins

two smaller arteries and one larger vein Explanation: When inspecting the vessels in the umbilical cord, the nurse should expect to encounter one larger vein and two smaller arteries. In 0.5% of births (3.5% of twin births), there is only one umbilical artery, which can be linked to cardiac or chromosomal abnormalities.

The nurse is preparing a 40-year-old pregnant client for an amniocentesis. When preparing the room with the equipment needed, which is most important for safety of the fetus? vital sign equipment ultrasound equipment external fetal heart monitor Foley catheter tray

ultrasound equipment Explanation: An amniocentesis is an invasive procedure where a syringe is used to remove a small amount of fluid for testing. All of the supplies and equipment listed may be necessary. Because the health care provider needs to find a pocket of fluid away from the fetal body, an ultrasound is the most important for fetal safety. An ultrasound allows a view of the inside of the placenta. The nurse should monitor fetal heart tones (FHTs) to ensure that the fetus is not in distress; the external fetal monitor is most often used. The nurse should also instruct the client to empty their bladder; less frequently, the client will have a Foley catheter placed before the test (to prevent bladder rupture). The nurse will monitor the client's vital signs during the test and for at least 1 hour afterward.

The nurse teaches a primigravida client that lightening occurs about 2 weeks before the onset of labor. What will the mother likely experience at that time? dysuria dyspnea constipation urinary frequency

urinary frequency Explanation: Lightening refers to the descent of the fetal head into the pelvis and engagement. With this descent, pressure on the diaphragm decreases, easing breathing, but pressure on the bladder increases, leading to urinary frequency. Dysuria might indicate a urinary tract infection. Constipation may occur throughout pregnancy due to decreased peristalsis, but it is unrelated to lightening.

A pregnant woman has decided that she wants to receive epidural analgesia for pain control during labor and birth. After teaching the woman about this type of pain control, the nurse determines that additional teaching is needed when the woman identifies which condition as being associated with epidural analgesia? hypotension headache urinary frequency itching

urinary frequency Explanation: Possible complications that can occur from epidural blocks include hypotension, pruritus (especially if morphine was used), urinary retention, nausea and vomiting, and rarely, a postdural puncture (spinal headache) because the subarachnoid space was entered for the analgesic injection.

A client is being prepared for a scheduled cesarean birth by the medical team. Which intervention will be most critical for the team to monitor in the first 24 hours post-cesarean birth? urinary output administration of sodium citrate preparing for blood transfusion maintaining IV access

urinary output Explanation: It will be critical to carefully monitor the client's kidney function by monitoring her urinary output. This may include maintaining the Foley catheter for the first 24 hours to ensure accurate measurements of urinary output. Sodium citrate may be administered before surgery to help minimize gastric upset. Maintaining the IV access will depend on the client's situation; however, it is not the priority that monitoring her kidney function is.

A fundal massage is sometimes performed on a postpartum woman. The nurse would perform this procedure to address which condition? uterine atony uterine prolapse uterine subinvolution uterine contraction

uterine atony Explanation: Fundal massage is performed for uterine atony, which is failure of the uterus to contract and retract after birth. The nurse would place the gloved dominant hand on the fundus and the gloved nondominant hand on the area just above the symphysis pubis. Using a circular motion, the nurse massages the fundus with the dominant hand. Then the nurse checks for firmness and, if firm, applies gentle downward pressure to express clots that may have accumulated. Finally, the nurse assists the woman with perineal care and applying a new perineal pad.

A woman with a history of crack cocaine use disorder is admitted to the labor and birth area. While caring for the client, the nurse notes a sudden onset of fetal bradycardia. Inspection of the abdomen reveals an irregular wall contour. The client also reports acute abdominal pain that is continuous. Which condition would the nurse suspect? amniotic fluid embolism shoulder dystocia uterine rupture umbilical cord prolapse

uterine rupture Explanation: Uterine rupture is associated with crack cocaine use disorder. Generally, the first and most reliable sign is sudden fetal distress accompanied by acute abdominal pain, vaginal bleeding, hematuria, irregular wall contour, and loss of station in the fetal presenting part. Amniotic fluid embolism often is manifested with a sudden onset of respiratory distress. Shoulder dystocia is noted when continued fetal descent is obstructed after the fetal head is delivered. Umbilical cord prolapse is noted as the protrusion of the cord alongside or ahead of the presenting part of the fetus.

The obstetric nurse is caring for a pregnant client who has been diagnosed with a hydatidiform mole. What assessment should the nurse prioritize? vaginal bleeding blood pressure pain severe nausea and vomiting

vaginal bleeding Explanation: Molar pregnancies constitute a major risk factor for vaginal bleeding. The client does not normally have an increased risk for nausea, pain, or hypertension.

As part of an in-service program to a group of home health care nurses who care for postpartum women, a nurse is describing postpartum depression. The nurse determines that the teaching was successful when the group identifies that this condition becomes evident at which time after birth of the newborn? in the first week within the first 2 weeks in approximately 1 month within the first 6 weeks

within the first 6 weeks Explanation: PPD usually has a gradual onset and becomes evident within the first 6 weeks postpartum. Postpartum blues typically manifests in the first week postpartum. Postpartum psychosis usually appears about 3 months after birth of the newborn.

The nurse reviews the newborn's morning laboratory levels and notes a bilirubin level of 5.8 mg/dl (99.20 µmol/l). What will the nurse expect to assess in the newborn? stools that are seedy and yellow yellow-tinted skin on the head and face yellowing of the soles of the feet enlarged liver, palpable on examination

yellow-tinted skin on the head and face Explanation: When the newborn's serum bilirubin level is 4 to 6 mg/dl (68.42 to 102.62 µmol/l) or higher, the newborn will exhibit jaundice. The appearance of jaundice (yellowing) will start on the head and face and move down the trunk. The stools of the newborn will not be yellow due to jaundice. The liver is overwhelmed by the large volume of red blood cells but is not physically enlarged.


Conjuntos de estudio relacionados

Chapter 6 Microbial Nutrition & Growth

View Set

Management Chapter 6- Strategic Management

View Set

FA #19 - Characteristics of Projectiles

View Set

Social Studies 2021-22 General terms

View Set

Psych 1X03: Instrumental Conditioning

View Set

example exam questions exam 2 (Q1-12)

View Set

World History Unit 5: Enlightenment

View Set

AP Calculus Review - Integration

View Set

economics chapter 7 practice test

View Set